Friday, August 29, 2008

29 August Gre Verbal Question

1.Gruff

2.Rivet

3.Pacify

4.Qurush

5. Repudiate

6. Complaisance

7. Encomium

8. Embezzle

9. Adamant

10. Demoralized

11. Natatorium

12. Imbue

13. Vapid

14. Clairvoyant

15. Obscene




ANALOGIES:


1.Game: Stadium

2.Brake: speed

3.Embezzling: stealing

4.Generator: Electricity

5.Factory: Product

29th august GRE Quants Question

1. If a/b @ c = (a/b) / c then (7/3) @ 4 =?

2. Col A: 100! / 99!
Col B: (100! – 99!) / 98!

3. Given three points a, b, c. If a, b lie on the same line and if all these three points are at same distance between each other, then
A. None of the points lie on the same plane.
B. Only one point lies on the plane.
C. Two points lie on the plane.
D. More than two points lie on the plane.
E. None of the above.

4. Given a series -9, 10, -11, 12, -13. . . . . . . . . . .80
Col A: sum of first 27 numbers
Col B: -22

5. If 100 < Y
Col A: 2y
Col B: 400

6.

As in above figure a, b and c are three angles & it is given that RQ is parallel to NO and PQ is parallel to MN.
Col A: a
Col B: b+c

Thursday, August 28, 2008

28 th August GRE Quats Question

1. If N is an odd and negative integer and it is a product of ten different integers.
Col A: no. of odd integers used in the product to get N
Col B: no. of negative integers used in the product to get N

2. Given a figure of combination of two triangles, in which first triangle contains 90, x, y degree angles and second triangle contains 90, z, w degrees. Which of the following conditions must be true?
I. x - w = z - y
II. x + y = w
III. z = 90 - w
A. I only
B. II only
C. I and III only
& so on....

3. Given a sequence -9, 10, -11, 12, -13, 14, -15..... Find the sum of first 40 integers in sequence?

4. If n > 1 then
Col A: (-1)^(4n-3)
Col B: (-1)^(4n-2)

5.

If all the seven circles are shaded like above then what is the area of unshaded region?

6. The standard deviation of the series 3, 7, 9, 13, 17 is?

7. Col A: 77!/78!
Col B: 87!/88!

8. If 2^n > 10^15n then which of the following options satisfies the condition
A. 30
B. 45
C. 60
D. 75

9. If x, y are the prime numbers greater than 10, then which of following must be even?
A. (x^2)*(y^2)
B. x^2 + x*y
C. 2*x*y + x
& so on....

10. Given a line in xy-plane with slope k = -3 & the points are (2, k), (-3, m)
Col A: k-m
Col B: -15

11. The approximate value of ((61.16)(.9Cool^2)/sqrt(401) is
A. 5
B. 4
C. 3
D. 2
E. 1

12. Col A: 0.9999/0.9998
Col B: 1.0002/1.0001

13. There is a junior and senior team who has taken tests. The average of junior team members who have taken tests are 88 and the average of senior team members is 92
Col A: the average of senior and junior team
Col B: 90

14. When a number is divided by 12, the remainder is 5. What is the remainder when the square of that number is divided by 8?

15. If the selling price of a product is 25% lesser than its list price and 40% greater than its cost price, then what is the list price of the product if the cost price = 30 ?

16. Last year, 1/4 of factory workers are architects. One year ago 60 workers newly joined in which 50 are architects and no one left the factory since last year and if now 1/3 of factory workers are architects then find the total number of workers?

17. When 'w' is divided by 14, the remainder is zero. If 'w' is three lesser than it value and when divided by 15, its remainder is 14. What is the value of 'w'?

18. If RST is isosceles triangle with RS = ST and P be a point on the side RT, then which of the following conditions must be true:
i) SP < ST
ii) SP < RT
iii)SP < PT

Monday, August 25, 2008

25th August Gre Verbal Question

Today's Words:


1. Exacerbate

2. Stern

3. Debauched

4. Lampoon

5. Impeachable

6. Gratuitous

7. Feral

8. Amenable

9. Futility

10. Pragmatic

11. Saboteur

12. Disrupt




Analogies:

1. Starryeyed: Pragmatic

2. Saboteur: Disrupt

3. choreograph:movement

25th August Gre Quants Question

If AB = BC, CI = IH & DE = EF then
Col A: Area of Rectangle GDEF
Col B: Sum of areas of Rectangle ABCD & CIHE

2. What is the closest integer to sqrt(171)?

3. If an = n { (-1)^n – 1)}, and a1, a2, a3, a4, a5 are the terms then what is the difference between greatest & least term?

4. No. of Members Least weight Range
20 members 100 29
21members 130 35
When two groups are joined, what is the median weight of the group?

5. Col A: 0.9999/0.9998
Col B: 1.0002/1.0001

6. If (2x+7) < 13 then
Col A: x^2
Col B: 9

7.

If AC = x, then find the perimeter of the figure?

Friday, August 22, 2008

Barrons Gre Flash Card

Just Take out the print out and fold the Card in appropriate manner so that u can hide the answer of gre word and check your answer by seeing back.

Read this document on Scribd: Barron FlashCard

22 august gre quats question

1. There is a junior and senior team who have taken tests. The average of junior team members who have taken tests are 88 and the average of senior team members is 92
Col A: the average of senior and junior team
Col B: 90

2. Given 0.61 Col A: square root(x)
Col B: 0.73

3. A toy manufturer company manufactures 20,000 toys and exports ¾ and 1/10 and donates some (xxx) value for charitable trust.
Find the number of toys left in the company?

4. There are rooms from 101 to 550 inclusive in which the starting rooms are numbered from 1, 2, 3 and the waiting rooms are numbered from 4,5,6. What is the probability that the rooms 4, 5, 6 will be choosed randomly?

5. One side of a triangle is given as 6 and the triangle is equilateral triangle & another triangle is joined with it whose side is 10
Col A: perimeter of polygon
Col B: 30
(Question is similar to this)

6. Given a line in xy plane with slope k = -1/2
Col A: x-intercept
Col B: y-intercept

7. If x < y then
Col A: x^2+x+1
Col B: y^2+y+1

8. Which of the following number is nearer to sqrt(171)?
A.12
B.13
C.14
D.15
E.16

9. When a number is divided by 12, the remainder is 5. What is the remainder when the square of that number is divided by 8?

10. The area of circle of radius 'r' is twice the the area of triangle then r in terms of h is?

11. Col A: 0.001^-1 + 0.999^-1
Col B: 0.002^-1 + 0.998^-1

12. If the selling price of a product is 25% lesser than its list price and 40% greater than its cost price, then what is the list price of the product if the cost price = 30 ?
A. 52
B. 53
C. 56
& so on.....

13. If x and y are prime numbers greater than 10 then which of the following is an even integer?
A. x^2+xy
B. x^y +2
C. xy +2
D. 2xy +xy

14. If z>1 & z(x+y+z) = zx+zy+z
Col A: z
Col B: 0

15. If x<2 & y<3
Col A: xy
Col B: 6

16. The average salary of n employees is 32000, average salary of another different group of n-1 employees is 33000
Col A: The average salary of 2n-1 employees
Col B: 32500

17. The range of 'p' numbers is 9.5 and the range for 'q' numbers is 21.5, what is the least range value when p and q are combined together?

18. A series was given as 1, -3, 5,-7 ,9,-11,13............ In this series odd numbers are negative integers. Find the sum of first 25 numbers in the series?

19. If n is an integer then
Col A: (-1)^6n+9
Col B: (-1)^2n-1

20. A trapezoid is given whose area is 12, length of it is given as 6 & height of it is given as 3. Find the perimeter of trapezoid?

Antonyms Test 2

ABRASION : STROKE ::

(A) boil : heat
(B) rubble : demolish
(C) flattery : compliment
(D) remnant : cut
(E) nut : shell


JUDGE : IMPARTIAL ::

(A) animal : tame
(B) acrobat : limber
(C) dignitary : proud
(D) prisoner : repentant
(E) politician : liberal


DRAWER : BUREAU ::

(A) tributary : river
(B) trunk : tree
(C) article : magazine
(D) kernel : husk
(E) language : dictionary


BATHE : WATER ::

(A) spread : knife
(B) fly : airplane
(C) obstruct : plug
(D) point : finger
(E) cover : coat


ILLICIT : RATIFY ::

(A) obsolete : preserve
(B) confusing : obscure
(C) popular : criticize
(D) insignificant : highlight
(E) belligerent : appease


PROTAGONIST : CHARACTER ::

(A) brush : applicator
(B) lawmaker : government
(C) costume : gala
(D) conductor : orchestra
(E) novice : competitor



CULTIVATE : GARDEN ::

(A) play : game
(B) worship : church
(C) explore : cave
(D) relax : vacation
(E) steer : automobile


EMPLOY : INDIGENCE ::

(A) expose : dormancy
(B) arrive : presence
(C) submerge : buoyancy
(D) season : blandness
(E) know : ignorance


SIEVE : DIVIDE ::

(A) staple : collate
(B) funeral : mourn
(C) siphon : squeeze
(D) panacea : remedy
(E) monarch : serve


MEDIATE : MEDDLE ::

(A) reason : think
(B) accumulate : hoard
(C) neglect : abandon
(D) surpass : cheat
(E) discourage : burde


1. MASON :: STONE

A. soldier : weapon
B. lawyer : law
C. blacksmith : forge
D. teacher : pupil
E. carpenter : wood

2. ARTICULATE :: SPEECH

A. predictable : event
B. coordinated : movement
C. dangerous : disease
D. active : thought
E. erratic : path

3. INCEPTION :: CONCLUSION

A. departure : arrival
B. culmination : upshot
C. refutation : approval
D. approach : return
E. escapade : punishment

4. SCINTILLATING :: DULLNESS

A. erudite : wisdom
B. desultory : error
C. boisterous : calm
D. cautious : restraint
E. exalted : elevation

5. ELUCIDATE :: CLARITY

A. envision : memory
B. aggravate : problem
C. conceal : oblivion
D. illuminate : light
E. mystify enlightenment

6. SHARD :: POTTERY

A. seed : flower
B. smoke : fire
C. dish : menu
D. chair : furniture
E. splinter : wood

7. REPEL :: LURE

A. dismount : devolve
B. abrogate : deny
C. abridge : shorten
D. enervate : weaken
E. miscarry : succeed

8. PENURY :: MONEY

A. starvation : sustenance
B. independence : freedom
C. infirmity : illness
D. reality : foresight
E. spontaneity : care

9. ARABLE :: LAND

A. impenetrable : jungle
B. navigable : waterway
C. fertile : fertilizer
D. shallow : pond
E. flat : field

10. ATTENUATE :: SIGNAL

A. exacerbate : problem
B. modify : accent
C. dampen : enthusiasm
D. elongate : line
E. dramatize : play

11. FURNACE :: SLAG

A. vegetable : garbage
B. factory : goods
C. fire : ashes
D. automobile : gasoline
E. silo : grain

12. PROSAIC :: MUNDANE

A. obdurate : foolish
B. ascetic : austere
C. clamorous : captive
D. loquacious : taciturn
E. peremptory : spontaneous

13. SALACIOUS :: WHOLESOME

A. religious : private
B. expensive : profligate
C. conservative : stoic
D. mendacious : truthful
E. fulsome : generous

14. PONDER :: PROBLEM

A. remove : doubt
B. capture : runaway
C. seize : time
D. ruminate : idea
E. curl : hair

15. HACKNEYED :: FRESHNESS

A. stale : porosity
B. facile : delicacy
C. ponderous : lightness
D. central : vitality
E. relevant : pertinence



1. ELECTED :: INAUGURATION

A. enrolled : graduation
B. condemned : execution
C. chosen : selection
D. gathered : exhibition
E. appointed : interview

2. DIVIDEND :: STOCKHOLDER

A. patent : inventor
B. royalty : author
C. wage : employer
D. interest : banker
E. investment : investor

3. ARCHIPELAGO :: ISLAND

A. village : hamlet
B. constellation : star
C. river : sea
D. finger : hand
E. tongue : mouth

4. PRECIS :: ELABORATION

A. comprehension : understanding
B. revision : correction
C. simplification : decoration
D. accuracy : abnormality
E. expurgation : distortion

5. MINCE :: WALK

A. bang : sound
B. wave : gesture
C. waltz : dance
D. simper : smile
E. hike : run

6. DISINTERESTED :: UNBIASED

A. indulgent : intolerant
B. exhausted : energetic
C. languid : lethargic
D. unconcerned : involved
E. profligate : flippant

7. AMBIVALENT :: CERTAIN

A. indifferent : biased
B. furtive : open
C. impecunious : voracious
D. discreet : careful
E. munificent : generous

8. DEFERENCE :: RESPECT

A. admiration : jealousy
B. condescension : hatred
C. affection : love
D. pretence : truth
E. gratitude : charity

9. AUTHORITARIAN :: LENIENT

A. philanthropist : generous
B. virtuoso : glamorous
C. hedonist : indulgent
D. servant : servile
E. miser : charitable

10. ALLAY :: SUSPICION

A. tend : plant
B. impede : anger
C. calm : fear
D. fell : tree
E. exacerbate : worry

11. PERENNIAL :: EPHEMERAL

A. volatile : evanescent
B. mature : ripe
C. diurnal : annual
D. permanent : temporary
E. majestic : mean

12. DIRECTORY :: ADDRESS

A. list : number
B. catalogue : shop
C. thesaurus : fact
D. dictionary : spelling
E. encyclopaedia : solecism

13. PRIMITIVE :: SOPHISTICATE

A. employee : superior
B. socialite : recluse
C. tyro : expert
D. native : inhabitant
E. applicant : member

14. TAP :: PUMMEL

A. tiptoe : stamp
B. hit : hurt
C. lumber : stumble
D. talk : chat
E. pump : water

15. TRAGEDY :: DRAMA

A. farce : actor
B. cartoon : film
C. prosody : poem
D. accident : ambulance
E. epigram : anecdote



BOAST :: LANGUAGE

A. glare : anger
B. swagger : gait
C. stare : sight
D. grow : height
E. enrage : anger

2. BELITTLE :: DISPARAGE

A. jeopardize : protect
B. efface : inscribe
C. assuage : increase
D. deride : ridicule
E. decrease : augment

3. VIRULENT :: INNOCUOUS

A. pretentious : harmful
B. reprehensible : praiseworthy
C. strong : delicate
D. epidemic : widespread
E. antiseptic : medical

4. INCOHERENT :: CLARITY

A. quiet : volume
B. normal : austerity
C. stagnant : light
D. scribbled : writing
E. tidy : mind

5. DESECRATE :: HOLY

A. despoil : beautiful
B. beautify : ugly
C. glamorize : rich
D. damage : corporeal
E. improve : dull

6. VIRTUOSO :: ACCOMPLISHED

A. aesthete : austere
B. servant : servile
C. hedonist : pretentious
D. priest : orthodox
E. philanthropist : generous

7. AUTHORITARIAN :: STRICT

A. fallacious : fraught
B. fastidious : particular
C. exemplary : arrogant
D. apprehensive : eager
E. neutral : objective

8. HOMOGENEOUS :: KIND

A. fast : speed
B. suspicious : origin
C. diverse : route
D. contemporary : time
E. disparate : place

9. CONSPICUOUS :: VISIBLE

A. sensual : audible
B. irrevocable : changed
C. elastic : stretched
D. ignominious : denounced
E. sensitive : felt

10. AUTHENTIC :: APOCRYPHAL

A. immune : dangerous
B. conventional : unorthodox
C. pious : religious
D. eccentric : strange
E. reticent : chaotic

11. TEMPER :: HARD

A. mitigate : severe
B. provoke : angry
C. endorse : tough
D. infer : certain
E. scrutinize : clear

12. STANZA :: POEM

A. chapter : novel
B. prose : verse
C. stave : music
D. song : chorus
E. overture : opera

13. ABSTEMIOUS :: INDULGE

A. subversive : undermine
B. diffident : confide
C. profligate : spend
D. gregarious : participate
E. submissive : assert

14. FRAGMENT :: MERGE

A. engage : marry
B. loose : tighten
C. splinter : join
D. mend : rend
E. diverge : convex

15. PROLIFIC :: UNFRUITFUL

A. new : old
B. blooming : withered
C. fertile : barren
D. fresh : stale
E. fecund : fervid

Antonyms Test -1

ZENITH

A apogee
B nadir
C collapse
D apotheosis
E denial

INGENUOUS

A naive
B insincere
C thrifty
D lachrymose
E innocuous

ATTENUATE

A mollify
B dilute
C enhance
D incapacitate
E distend

SYCOPHANT

A cynic
B critic
C philanderer
D philanthropist
E follower

RETICENCE

A candor
B silence
C denunciation
D powerlessness
E gentility

IMPERIOUS

A diffident
B gregarious
C egregious
D facetious
E overbearing

PROBITY

A rectitude
B infidelity
C corruptibility
D infamy
E uprightness


SANGUINE

A perturbed
B pedantic
C trite
D pessimistic
E defective


BUCOLIC

A pastoral
B urban
C idyllic
D chimerical
E chromatic


FRACTIOUS

A tacit
B brittle
C docile
D durable
E delicate

EFFULGENT

A concise
B drab
C vivacious
D terse
E sane


ABLUTION

A pollution
B lavation
C fumigation
D debasement
E decompositon


OBSTREPEROUS

A inchoate
B mundane
C compliant
D celebrated
E messy


SCURRILOUS

A fastidious
B scandalous
C decent
D voluptuous
E evocative

INVETERATE

A decrepit
B habitual
C chronic
D conditional
E novel

PICAYUNE

A grandiose
B extravagant
C trivial
D magnanimous
E immense


VALUE

A assess
B appreciate
C misestimate
D depreciate
E disdain


LETHARGY

A excitement
B torpor
C delirium
D drowsiness
E verve

SALUBRIOUS

A detrimental
B mendacious
C indignant
D incorruptible
E elevated

DILATORY

A serious
B prompt
C undefeated
D jovial
E indolent

INAUGURAL

A initial
B ultimate
C penultimate
D conterminous
E determinate


PERSPICACIOUS

A beautiful
B domineering
C unobservant
D dynamic
E persuasive


MERCURIAL

A staid
B capricious
C dedicated
D irascible
E simple


SWAY

A power
B control
C impotence
D insignificance
E illegitimacy


HALE

A infirm
B healthy
C cancerous
D injured
E vigorous


SOPORIFIC

A analgesic
B mature
C callous
D arousing
E emphatic


QUIXOTIC

A timid
B deleterious
C inoffensive
D pragmatic
E erratic

MAWKISH

A arrogant
B divisive
C creative
D indecisive
E unsentimental

PARSIMONIOUS

A feral
B cordial
C affable
D prodigal
E irascible

VOLUBLE

A glib
B infinitesimal
C taciturn
D boisterous
E frigid


SYBARITE

A hedonist
B accomplice
C protegee
D friend
E ascetic

PHLEGMATIC

A excitable
B dogmatic
C destructive
D ineluctable
E handsome

TACT

A imprudence
B misconduct
C indiscretion
D finesse
E thoughtlessness

DEFILE

A purify
B rectify
C infuse
D organize
E restore


SURREPTITIOUS

A clandestine
B overt
C covert
D stealthy
E rife

INTRANSIGENT

A passive
B complacent
C meek
D conciliatory
E forward

APPROBATION

A commendation
B liberation
C qualification
D alliance
E disapproval

SAGACITY

A illness
B convalescence
C vaccination
D inexperience
E ebullience

MALLEABLE

A docile
B intractable
C derogatory
D impressionable
E voracious


CAPACIOUS

A unfettered
B cramped
C deleterious
D callous
E egregious


APHASIA

pain
illness
solid state
speech loss
volubility


BESTIAL


noble
zoological
excellent
fuzzy
poor

ANTIPATHY

willingness
hatred
fondness
unwillingness
misery


ANTITHESIS

resurrection
destruction
opposite
similarity
creation

AMBULATORY

bedridden
evolutionary
emerging
emergency
congenital

BERATE

praise
accept
refer
deny
purchase


CHASTE

bold
wanton
hazy
followed
clear

CHIDE

unite
avoid
praise
help
follow

BIGOTRY

monogamy
arrogance
mourning
tolerance
polygam

BAROQUE

loud
simple
criticize
clumsy
monologue


CHURLISH


polite
religious
upset
rude
compressed


CAPRICIOUS


scattered
greedy
satisfied
steadfast
phenomenal


CARTE BLANCHE


restrictions
freedom
socialistic
commumal
capitalism

AMALGAMATE


conglomerate
separate
mixture
responsible
correspond

AMELIORATE


inflation
deflation
improve
recuperate
make worse

AMBIGUOUS


salvagable
hazy
clear
fuzzy
ancillary



BIZARRE


strange
normal
extol
staged
triumphant

CAPTIOUS


ambitious
capable
tolerant
humorous
incapable

ANONYMOUS


signed
expert
saint
unknown
subtle

ANTEDILUVIAN


ancient
ruins
old
modern
remains


# ABOMINATE :

1. loathe
2. despise
3. adore
4. abhor
5. attach


# OBSEQUIOUS :

1. servile
2. first
3. fawning
4. supercilious
5. improper


# OROTUND :

1. not resonant
2. not reddish
3. not eager
4. pompous
5. loud


# RECANT :

1. entangle
2. rescue
3. fail
4. assert
5. predict


# UPBRAID :

1. defer
2. vacillate
3. sever
4. conjoin
5. laud


# PLENITUDE :

1. luxury
2. magnificence
3. richness
4. contentment
5. scarcity


# SCURRILOUS :

1. decent
2. savage
3. major
4. volatile
5. scabby


# FULMINATION :

1. praise
2. repetition
3. escape
4. ratification
5. addition


# DISTEND

1. deteriorate
2. weaken
3. constrict
4. concentrate
5. fold


# TOUT

1. cast aspersions on
2. deny the relevance of
3. placate
4. withhold consent
5. misrepresent

# SQUALID

1. fervid
2. florid
3. pristine
4. extraneous
5. abundant


# SCOTCH

1. renovate
2. entrust
3. unfasten
4. encourage
5. emphasize


# PERFIDY

1. tact
2. generosity
3. thoroughness
4. loyalty
5. gratitude


# OUTLANDISH

1. conventional
2. prolific
3. unchanging
4. transparent
5. noticeable


# PLUMB

1. reversed
2. lofty
3. horizontal
4. thin
5. light


# FERVID

1. undistinguished
2. unexpected
3. stubborn
4. restrained
5. discouraged


# VACUITY

1. quality
2. certainty
3. plenitude
4. stability
5. incontinence


# RAVEL

1. knit
2. omit
3. remain silent
4. measure
5. increase in value


# PERSISTENCE

1. irrelevance
2. inconstancy
3. inequality
4. intemperance
5. incompetence.


# SUBROSA

1. openly
2. fashionably
3. under the owse
4. simply
5. clandestinely

General Strategies for Reading Comprehension

General Strategies for Reading Comprehension

1. Try to read the whole text of the passage once, if possible. Many people think you should just skim the passage or read the first lines of every paragraph, and not to read the passage. We believe this is an error: if you misunderstand the main idea of the passage, you will certainly get at least some of the questions wrong. Give the passage one good read, taking no more than 3 minutes to read all of the text. Do not read the passage more than once – that wastes too much time. If you have not understood it completely, try to answer the questions anyway. Note: this point of reading the whole passage is important for test-takers whose first language is not English, provided that they can read the passage in 3 minutes or less.

2. Make brief notes on the text on your scrap paper. As we will see below in greater detail, you should write down a couple of words on A) the Main Idea or Primary Purpose, B) Organization/Structure of the passage, and C) the Tone or Attitude of the author (if applicable). You just need a few words for each of these areas, and altogether it should not take longer than 30 seconds to write down.

3. Remember that the tone or attitude of the passage is usually respectful and moderate, never going to extremes of praise nor criticism. ETS obtains its Reading Comprehension passages from real articles about real academics and professionals. So the tone of the articles, even when there is criticism in the passage toward an academic or her work, is always balanced and moderate. In the same vein, articles that deal with minorities or ethnic groups are almost always positive and sympathetic.

4. Look out for structural words that tell you the important ideas or transitions in a passage.

Continue the Idea Words
Similarly
Moreover
Additionally
In the same way
Likewise

Conclusion Words
Thus
Therefore
Hence
So
In summary
In conclusion

Contradiction or Contrast Words
Neverthless
Nonetheless
However
But
Although
Though
Even though
Notwithstanding
Yet
Despite
In spite of
On the one hand…on the other hand
While
Unlike

5. Go back to the text of the passage for the answers. Many test-takers fail to return to the text of the passage to look for the correct answers. They rely solely on their memories and understanding of the passage after having read or skimmed it. Wrong. ETS is counting on that. Go back to the text to look for information to answer the questions. Nine times out of ten, the answer lies within the passage.

Of the 6 most important types of questions for Reading Comprehension, we will first look at Main Idea/Primary Purpose Questions, and the strategies we can use to answer them.
Main Idea/Primary Purpose Questions
Many people believe there is no difference between the main or central idea of the passage and the primary purpose of the author of the passage. This is simply not true. Let's take a look at the subtle but important difference between them:
Main Idea
The question might look something like this:

"Which of the following best states the central idea of the passage?"
"Which of the following most accurately states the main idea of the passage?"
"Which of the following is the principal topic of the passage?"
"The main topic of the passage is...."

Primary Purpose
The question might look like this:

"The primary purpose of this passage is to..."
"The primary purpose of the passage as a whole is to..."
"The primary focus of this passage is on which of the following?"
"The main concern of the passage is to..."
"In the passage, the author is primarily interested in...."
"The passage is chiefly concerned with..."
Strategy:
Main Idea: Look in the first and last paragraphs for the main idea. Any conclusion words like therefore, thus, so, hence, etc. that you see are most likely introducing the main idea. The correct answer will say the same thing as it says in the text, but using different words. The Main Idea is not always stated explicitly in the passage – in fact, more likely than not, it is not stated explicitly. Therefore, in order to answer this type of question when it is more implicit:

1. Re-read the first line of every passage, and the last line of the first and last paragraphs. This should give you the general structure or outline of the argument, with which you can answer the Main Idea question.
2. After determining the general structure or content of the argument, eliminate answer choices that are too broad or too specific, i.e. answer choices that go beyond the content of the passage, or that deal with content only discussed in one paragraph of the passage.
3. Make brief notes – a couple of words- regarding the Main Idea on the text on your scrap paper while you read.
Primary Purpose: What is the author trying to do? What is his intention? If he is evaluating a theory, then the answer could be something like "Discuss an interpretation". Note that the correct answer would deal with "an interpretation", because the author is only dealing with one theory. If the Primary Purpose is to criticize 2 new books, then his intention or his primary purpose might be to "Critique new studies". Again, as in Main Idea questions, re-read the first line of every passage, and the last line of the first and last paragraphs. This should give you the general structure or outline of the argument, with which you can answer the Primary Purpose question.
Note: A good main idea or primary purpose does not go beyond the scope of the passage, nor does it limit itself to discussing only one part of the passage.


What is the primary purpose of this passage?

A)
discuss the importance of the television program Star Trek for the international space program
B) discuss important theoretical work concerned with faster-than-light space travel.
C) explore a dispute among theoretical physicists regarding the uses of space flight
D) describe the possible uses of space-warping material
E) explain how a space-warping bubble would work in the real world

Explanation
This is a Primary Purpose question, so we have to determine what the author is trying to do or say in this passage. So, let's read the first and last lines of the passage in order to get an idea of the primary purpose. The first line says "Great news for Star Trek fans: warp drives that can propel starships around the Galaxy faster than the speed of light may be possible after all--with a little help from Dr Who." The last line is a quote by a physicist that says "Of course, there are still some basic questions--like how does one go about constructing this Tardis space-time--but it puts the concept of space warps back on the agenda." From both these sentences, we get the idea of space travel, faster than light travel and space warps – maybe this is a discussion of faster than light space travel. Does that match what you have already read? Yes, basically this is a discussion of the theoretical state of play in the area of faster-than-light space travel. Do any of the 5 answer choices match that? Yes – B, even if the wording is somewhat different from how we are wording it, the idea is almost exactly the same. B is the answer.
Another way of getting to the answer is through elimination of obviously incorrect answer choices. We can eliminate A because the author mentions the popular science fiction program Star Trek merely to introduce the idea of faster-than-light travel, and nothing more. C is a stronger possibility because the second paragraph of the passage does discuss some disagreement among physicists about the possibility of creating a warp-drive, but in the same paragraph the theoretical dilemma seems resolved. Moreover, since the author only discusses this in one paragraph, it cannot be the primary purpose of the entire passage. We can eliminate D because the author does not go into detail discussing the uses of space-warping material. And we can discard E because the author does not really go into how the space-warping bubble would work in the real world.

Title Questions
by Stephen Bolton, 20th August, 1999
Title questions are very similar to Main Idea questions, though are less common. Though some of the example paassage we use in this tutorial and in the Practice Section are from the New Scientist, and therefore have titles, the passages in the real GMAT will not have titles. The question might look like this:

"Which of the following titles best summarizes the passage as a whole?"
Strategy:
Treat this as a Main Idea question. A good title sums up the central idea of a passage. Therefore, in order to answer this type of question:

1. Look in the first and last paragraphs for the main idea. Any conclusion words like therefore, thus, so, hence, etc. that you see are most likely introducing the Main Idea/Title. The correct answer will say the same thing as it says in the text, but using different words.
2. Re-read the first line of every passage, and the last line of the first and last paragraphs. This should give you the general structure or outline of the argument, with which you can answer the Title question.
3. Make brief notes – a couple of words- regarding the Title on the text on your scrap paper while you read.
4. After determining the general structure or content of the argument, eliminate answer choices that are too broad or too specific, i.e. answer choices that go beyond the content of the passage, or that deal with content only discussed in one paragraph of the passage.

What would be an appropriate title for this passage?

A)
Constructing The Tardis
B) How To Make Space-Warping Material
C) Bubbles In Space-Time
D) Faster-Than-Light Travel: A Possibility?
E) Debate On The Uses of Space Travel

Explanation
This passage actually already has a title, "Warp Factor One". But we have to look for another title possibility, one that would be most like the Main Idea of the passage. We look at the first and last paragraphs, and since the Main Idea is that researchers now feel that faster-than-light travel maybe more than mere fantasy, we can find the correct answer choice. Does any answer choice corrspond to this idea? Yes- answer D, which is the correct answer.
We can also find the correct answer through elimination. There is nowhere in the passage where it discusses building Dr. Who's Tardis (pity!), so we can eliminate A. Nor does it tell us how to make space-warping material. Eliminate B. While bubbles in space-time are discussed at some length in one of the paragraphs, we cannot say this is the main concern of the passage, and thus should eliminate C. And nowhere are the uses of space travel discussed, so discard E.

Specific Detail or Target questions are probably the most common types of questions, and the easiest to answer. The question might look like this:

"According to the passage,...."
"The passage states that ...."
Strategy
The Specific Detail or Target that we are looking for could be a Line Number, or a Name or Date. Go to the Line Number or Name or Date, and then read several lines above and below it. Find the answer choice that basically says the same thing as in the passage, though usually with different words or word order.


According to the passage, Pfenning and Ford

A)
demonstrated conclusively the impossibility of faster-than-light travel
B) explored the possibility of bubbles that warp space
C) supported the work of Alcubierre
D) work at of the Institute for Theoretical Physics at the Catholic University of Leuven
E) suggested that a warp drive was not physically possible

Explanation
This is a Specific Detail/Target question, and therefore we look for the Name, Line Number, or Date that will help us. In this case, the detail consists of the names Pfenning and Ford. We scan the text, starting from the top of the passage, looking for the names Pfenning and Ford. We find them in only place, at the beginning of the second paragraph. We read a couple of lines above the names, and keep reading until a few lines after the names. It says "But in 1997 Michael Pfenning and Larry Ford at Tufts University in Medford, Massachusetts, apparently killed this ingenious idea by showing that it needed far more than the entire energy content of the Universe to work (This Week, 26 July 1997, p 6)". The line after that says the research of another physicist then resurrected the possibility of FTL travel, negating the implications of the research of Pfenning and Ford. Now we can answer the question. Do any of the answer choices match the information given around the target area? Yes- E.
Let's also eliminate. If we re-read what the passage says about Pfenning and Ford, we can eliminate B, C, and D. None of them are supported by the information in the passage, so let's eliminate all of them without wasting too much time and with a minimum of fuss. A is tougher to eliminate. From the sentence that mention Pfenning and Ford, it seems their work does rule out the possibility of a space-warp drive. But if we read the next line, it says another researcher said it was indeed possible. So the Pfenning and Ford could not have "conclusively" demonstrated the impossibity of the FTL drive.



This is probably the most difficult type of Reading Comprehension problem. The question might look like this:

"It can be inferred that the author makes which of the following assumptions?"
"Which is an assumption underlying the last sentence of the passage?"
"Which of the following, if true, would most strengthen the hypothesis mentioned in lines 17-19?"
"With which of the following statements regarding chaos theory would the author be most likely to agree?"
Strategy:
1. First, treat this type of problem as a Specific Target question. Look for a target in the question, find it in the text, and then look above and below it. Often you do not have to infer very much, the answer remains within the text.
2. If the answer must be inferred and is not stated explicitly within the text, then choose the answer choice that can be inferred or assumed from the information given. Again, you should not have to infer very much – only one or two logical steps removed from the information in the passage.
3. Make sure that the answer choice you decide on does not violate or contradict the Main Idea of the passage - if it does, the answer choice is probably wrong.

It can be inferred that a house with the propeties of the bubble mentioned in the passage

A)
would be larger on the inside than on the outside
B) could move faster than the speed of light
C) might be very energy efficient
D) could move through time
E) would eventually fold in on itself and be destroyed

Explanation
First, let's try to deal with this question as a Specific Target problem. Is there a target in the question? Yes – the bubble. The bubble is first mentioned at the end of the second paragraph, and then discussed at length throughout the third paragraph. Remember, we have to look above and below that target area (as well as read the target area again), so quickly go through the second, third, and first part of the fourth paragraph.
When you are finished, look at the answer choices. Can any of them be inferred from the information given in the target area? Well, we could eliminate C, D, and E for simply not being supported by the information given in the passage. B – maybe, but a house moving through time seems pretty silly. But in the fourth paragraph the author talks about the Tardis, "which looked like a police box but had a spacious interior". Big on the inside, small on the outside. Is that like our house? Yes- answer A. As well, we can choose A because it does not go against or contradict the Main Idea in this case, which if it had, would have made it necessary to eliminate. So choose A.


The question might look like this:

"The author's attitude towards Morgan's theory could best be described as one of ..."
Strategy:
Look for descriptive words, adjectives or adverbs, that could tell you the author's attitude. For example, the words unfortunately or flaw suggest a negative connotation, while strength or valuable emphasize the positive. Make brief notes – a couple of words- regarding the Tone of the text on your scrap paper while you read. Additionally, keep in mind that the author's attitude toward a theory, book, or ethnic group will almost always be respectful, even when somewhat critical.


The author's attitude towards Miguel Alcibierre's theory could best be described as one of
A)
admiration
B) mild skepticism
C) unbridled scorn
D) dismay
E) complete objectivity

Explanation
Since this is a Tone/Attitude question, we must look in the passage for descriptive words that tell us what the author thinks of Alcibierre and his theory. In the second paragraph the author call's Alcibierre's theory "this ingenious idea". This is positive, and the only positive answer choice is A. A is the correct answer. As well, if we could not find the tone so easily, we could also eliminate C and D at the very least, for being too extreme.


The question might look like this:

"Which of the following best describes the organization of the passage?"
"Which of the following best describes the organization of the first paragraph of the passage?"
"One function of the third paragraph is to...."
Strategy:
Re-read the first line of every passage, and the last line of the first and last paragraphs. This should give you the general structure or outline of the argument, with which you can answer the question. Remember to make brief notes about the structure of the text on your scrap paper. If you are looking for the organization of one paragraph, read the first and second sentence of the paragraph. That will give you a rough idea of what is the structure or organization of the paragraph.
Which of the following best describes the organization of the second paragraph of the passage?
A)
Two investigations that support Alcubierre's theory are introduced
B) Possible objections to the uses of the warp drive are present, and then refuted
C) An objection to the practicality of the theory is raised, and then another work is cited to shore up the applicability of the original theory
D) A work of theoretical physics that supports Alcubierre's theory is raised, and then another that refutes it is presented
E) Alcubierre's theory is analyzed by a panel of several eminent physicists
Explanation
Read the first sentence of the paragraph: "But in 1997 Michael Pfenning and Larry Ford at Tufts University in Medford, Massachusetts, apparently killed this ingenious idea by showing that it needed far more than the entire energy content of the Universe to work (This Week, 26 July 1997, p 6)". Then read the second sentence: "Now Chris Van Den Broeck of the Institute for Theoretical Physics at the Catholic University of Leuven, Belgium, has resurrected Alcubierre's proposal". So if we out those two sentences together, and in different words, first the usefulness of Alcubierre's theory is questioned by two researchers, then the theory is validated by yet another researcher. Which of the answer choices is closest to this? C. None of the other answer choices follow the organizational pattern of the paragraph – they reverse it, or are completely dissimilar. C is the only possible answer.


1. Read the whole text of the passage once.
2. Make brief notes about the text on your scrap paper.
3. Remember that the tone or attitude of the passage is usually respectful and moderate, never going to extremes of praise nor criticism.
4. Look out for structural words that tell you the important ideas or transitions in a passage.
5. Go back to the text of the passage for the answers to specific questions.

Reading Comprehension - Test -2

Reading Comprehension

Practice Test No: 02

10-12 minutes

Should we really care for the greatest actors of the past could
we have them before us? Should we find them too different from
our accent of thought, of feeling, of speech, in a thousand minute
particulars which are of the essence of all three? Dr. Doran's
5 long and interesting records of the triumphs of Garrick, and other
less familiar, but in their day hardly less astonishing, players,
do not relieve one of the doubt. Garrick himself, as sometimes
happens with people who have been the subject of much anecdote
and other conversation, here as elsewhere, bears no very distinct
10 figure. One hardly sees the wood for the trees. On the other hand,
the account of Betterton, "perhaps the greatest of English
actors," is delightfully fresh. That intimate friend of Dryden,
Tillatson, Pope, who executed a copy of the actor's portrait by
Kneller which is still extant, was worthy of their friendship;
15 his career brings out the best elements in stage life. The stage
in these volumes presents itself indeed not merely as a mirror of
life, but as an illustration of the utmost intensity of life, in
the fortunes and characters of the players. Ups and downs,
generosity, dark fates, the most delicate goodness, have nowhere
20 been more prominent than in the private existence of those devoted
to the public mimicry of men and women. Contact with the stage,
almost throughout its history, presents itself as a kind of
touchstone, to bring out the bizarrerie, the theatrical tricks
and contrasts, of the actual world.

1. In the expression “One hardly sees the wood for the trees”, the author apparently intends the word trees to be analogous to

A. features of Doran’s language style
B. details learned from oral sources
C. personality of a famous actor
D. detail’s of Garrick’s life
E. stage triumphs of an astonishing player

2. The doubt referred to in line 7 concerns whether

A. the stage personalities of the past would appeal on a personal level to people like the author
B. their contemporaries would have understood famous actors
C. the acting of famous stage personalities would appeal to us today
D. Garrick was as great as he is portrayed
E. historical records can reveal personality

3. Information supplied in the passage is sufficient to answer which of the following questions?

I Who did Doran think was probably the best English actor?
II What did Doran think of Garrick?
III Would the author give a definite answer to the first question posed in the passage?

A. I only
B. II only
C. I and III only
D. II and III only
E. I, II and III





A sanctuary may be defined as a place where Man is passive and
the rest of Nature active. Till quite recently Nature had her
own sanctuaries, where man either did not go at all or only as
a tool-using animal in comparatively small numbers. But now, in
5 this machinery age, there is no place left where man cannot go
with overwhelming forces at his command. He can strangle to
death all the nobler wild life in the world to-day. To-morrow
he certainly will have done so, unless he exercises due
foresight and self-control in the mean time.

10 There is not the slightest doubt that birds and mammals are
now being killed off much faster than they can breed. And it
is always the largest and noblest forms of life that suffer
most. The whales and elephants, lions and eagles, go. The rats
and flies, and all mean parasites, remain. This is inevitable
15 in certain cases. But it is wanton killing off that I am
speaking of to-night. Civilized man begins by destroying
the very forms of wild life he learns to appreciate most when
he becomes still more civilized. The obvious remedy is to begin
conservation at an earlier stage, when it is easier and better
20 in every way, by enforcing laws for close seasons, game preserves,
the selective protection of certain species, and sanctuaries.

I have just defined a sanctuary as a place where man is passive
and the rest of Nature active. But this general definition is too
absolute for any special case. The mere fact that man has to
25 protect a sanctuary does away with his purely passive attitude.
Then, he can be beneficially active by destroying pests and
parasites, like bot-flies or mosquitoes, and by finding antidotes
for diseases like the epidemic which periodically kills off the
rabbits and thus starves many of the carnivora to death. But,
30 except in cases where experiment has proved his intervention to
be beneficial, the less he upsets the balance of Nature the
better, even when he tries to be an earthly Providence.

4. The author implies that his first definition of a sanctuary is

A. totally wrong
B. somewhat idealistic
C. unhelpful
D. indefensible
E. immutable

5. The author’s argument that destroying bot-flies and mosquitoes would be a beneficial action is most weakened by all of the following except

A. parasites have an important role to play in the regulation of populations
B. the elimination of any species can have unpredictable effects on the balance of nature
C. the pests themselves are part of the food chain
D. these insects have been introduced to the area by human activities
E. elimination of these insects would require the use of insecticides that kill a wide range of insects

6. It can be inferred that the passage is

A. part of an article in a scientific journal
B. extracted from the minutes of a nature club
C. part of a speech delivered to an educated audience
D. a speech delivered in a court of law
E. from a polemical article published in a magazine

7. The purpose of the final paragraph is

A. to sum up the main points of the author’s argument
B. to urge a solution to an increasingly pressing problem
C. to qualify the author’s definition of an important term
D. to propose a program
E. to suggest that man should not intervene in natural environments

Reading Comprehension Test - 1

Much has been made of the wealth-creating power of the New Economy. Technological innovation and entrepreneurial activity, proponents of the New Economy point out, have combined to spur the most prolonged economic expansion in American history, and have swelled the ranks of the upper middle classes. A recent Conference Board study, however, suggests that the economic boom of the 1990s has bypassed the poorest segments of the American population. The study reveals that since 1986, a period in which the American economy has expanded by some 30%, the number of fully employed people living in poverty has grown by some 40%, to nearly 3 million Americans.

Apologists of the New Economy are quick to downplay the significance of this disturbing trend. In the second half of the 1990s, they argue, the plight of the poor has begun to improve. Unemployment rates are at a post-war low, the real wages of low-income workers have edged upwards, and from 1993 to 1998 the poverty rate declined from 15.1% to 12.7%. These modest gains notwithstanding, there is no denying that for the economically disadvantaged the current economic boom has not brought the benefits that the boom of the 1960s did: from 1966 to 1978, for example, the poverty rate among full-time workers fell by more than 50%. There can only be one explanation for the fact that the recent boom has not produced similar effects. While the computerization of the American economy has created many high-skilled jobs in the technology industry, a vast number of the new jobs created in the current boom are of the low-skill and low-pay variety. To the people filling these jobs the New Economy has hardly yielded a rich bounty.


In paragraph two, the "apologists" are most characterized by their:

A role in promoting the rights of those on welfare
B regret at missing out on the New Economy
C sorrowful demeanor
D faith in the positive impact of the New Economy
E contrition with regard to poverty in America


According to the passage, what is the principal shortcoming of the New Economy?

A It has not lowered unemployment rates.
B The technological revolution has impersonalized society.
C It requires too much education to succeed in the age of the New Economy.
D It has not in fact increased the overall wealth of society as much as is often assumed.
E Many of the jobs it has created do not pay a good wage.

Which of the following is the most likely occupation of the author of the passage?

A A speechwriter for a politician
B A lobbyist for the computer industry
C A journalist for an investment magazine
D An advocate for the poor
E A disgruntled employee of the Conference Board





#2

The evolution of painting, and cubism in particular, shared with science the common characteristic of drawing upon late nineteenth-century achievements, but, in so doing, of intensifying and transforming them. The result was the overthrow of much of the heritage of the nineteenth and earlier centuries. In certain respects cubism brought to an end artistic traditions that had begun as early as the fifteenth century. At the same time, the cubists created a new artistic tradition that is still alive, for they originated attitudes and ideas that spread rapidly to other areas of culture and that to an important degree underlie artistic thought even today. Cubism first posed, in works of the highest artistic quality, many of the fundamental questions that were to preoccupy artists during the first half of the twentieth century; the historical and aesthetic importance of cubism, therefore, renders it worthy of the most serious attention.

Cubism developed with extraordinary rapidity between the years 1907 and 1914. From 1914 until about 1925 there were a great many artists painting in a cubist mode, but this later phase produced relatively few stylistic innovations that had not been anticipated to some extent during the pre-war years. By the mid-1920s, a crisis emerged in cubism as in European art generally, bringing to an end a period of almost twenty years during which cubism had been the predominant force behind an entire artistic generation.

In its beginnings, however, and until about 1923, cubism was an exclusively Parisian phenomenon, and it probably could not have been born elsewhere, for reasons of history, geography, and culture. No other city in the world in the early years of the twentieth century could boast of a comparable century-long history of outstanding artistic activity; and the relatively central location of Paris in western Europe served only to facilitate the migration of the most gifted young artists and writers from Spain, Italy, Germany, Russia, and the Low Countries toward this cultural mecca. Paris offered them not only the challenge of their most gifted contemporaries, but also its great museums; it offered a tradition of moral and intellectual freedom, and an artistic bohemia in which they could live cheaply at the edge of society without suffering the ostracism inflicted by the bourgeoisie in smaller, more conservative, and less cosmopolitan European cities. In retrospect it is not surprising that, by the early part of the twentieth century, Paris contained an astonishing number of young men of genius, whose presence constituted an intellectual ‘critical mass’ that soon produced a series of revolutionary cultural explosions.

Which of the following was NOT a reason given by the author that Paris became the center of the artistic world in the early twentieth century?

A Paris was centrally located in Western Europe
B Artists were attracted to Paris because of its many museums
C Parisian society was characterized by greater freedom than other European cities
D The bourgeoisie of Paris were wealthy and provided a vast market for young artists to sell their work
E Paris had a century-long tradition of outstanding artistic activity


Which of the following best summarizes the author’s view of the significance of cubism?

A Cubism was a revolutionary movement that transformed art and has continued to influence art up to the present.
B Cubism was a revolutionary movement that transformed art in the early twentieth century but exercised little
influence after the movement waned in the 1920s.
C Cubism, though an important movement, never exercised much influence outside of Paris.
D Cubism was a short-lived fad and doesn’t deserve serious attention from art critics or art historians.
E Cubism inaugurated a social revolution and had far-reaching effects that were felt far beyond the confines of the art world

The word "ostracism" in the third paragraph most nearly means:

A stigmatization
B gentrification
C calumniation
D poverty
E incrimination


Based on this passage it can be concluded that the author believes that:

A Cubism never exercised much influence outside the city of Paris
B The most innovative period of the cubist movement occurred while Cubism was confined to Paris
C Picasso was the greatest of all cubists
D The crisis that emerged in cubism in the 1920s caused a crisis in the rest of the art world
E Cubism had only an ephemeral impact on art


In the third paragraph the artist describes Paris as a "cultural mecca" because:

A he means to suggest that Paris was greatly influenced by Middle Eastern trends
B he believes that Paris’ importance as a religious center resulted in great amounts of artistic patronage
C he believes that Paris’ age-old importance as a center of European art was waning
D he means to suggest that Paris represented a place to which artists flocked from all over Europe
E he means to suggest that the Parisian art world had become decadent



# 3


Ichneumon wasps are a family (Ichneumonidae) of insects in the order Hymenoptera. While the life cycle of most ichneumons is still largely undocumented, observation of some of the known species of the order enables it to relate some established facts. Most ichneumons overwinter in the pupal stage, although a few species overwinter as adult fertilized females. In spring, after emerging from their pupae, the adults mate; the females of the species seek out other insects in which to lay their eggs. Unlike other members of the order Hymenoptera, which includes bees and ants as well as wasps, almost all species of ichneumon wasps are parasitic. Most parasitize only one species; a particular favorite is the moth or butterfly caterpillar. Virtually all the life cycle - barring the adult stage - takes place inside other insects, making observation difficult. It seems, however, that most eggs hatch while the host is still alive; wasp larvae usually live inside the host without killing it. Occasionally, the wasp begins its own development in one stage of its host's own life cycle, and will not mature until the host itself has transformed into another stage.

Adult ichneumons may be observed moving about flowers and leaves, feeling these with their antennae, searching for the prey on which they will parasitize. Many of the larger ichneumons lay their eggs on caterpillars and in the cocoons of large silk moths. If an insect is living and active, one cannot determine by observation alone whether it has been parasitized. However, dryness and brittleness of dead caterpillars and cocoons are often the signs that these had served as hosts to an ichneumon. The presence of small, hard, oval pupal cases inside the remains of the caterpillar or cocoon will corroborate these signs.

Megarhyssa macrurus, one of the largest ichneumons, lays its eggs in the developing larva of the horntail, a primitive wasp whose larva feeds in tunnels inside wood. The female megarhyssa uses her antennae to sense the vibrations of the horntail larvae inside the wood; her ovipositor penetrates through the wood, allowing her to lay her eggs in the horntail larvae. The eggs develop in the larvae, killing them when they are fully grown. The wasp pupates in the horntail tunnel, and chews its way through the bark upon maturity.

The primary purpose of this passage is to:

A Catalogue the destructive behavior of ichneumons.
B Explain what parasitical activity entails.
C Contrast the ways in which ichneumons differ from other hymenoptera.
D Detail what facts are known about the life cycle of ichneumons.
E Register the horntail, silk moth and butterfly among the targets of the ichneumon.


The passage says that ichneumons parasitize which of the following:
I Butterfly caterpillar
II Cocoons of silk moths
III Other wasps


A I only
B II only
C III only
D I & II only
E I, II & III


According to the passage above, the order Hymenoptera includes which of the following:
I Horntail wasps
II Parasitic bees
III All ants


A I only
B I and II
C I, II and III
D II and III only
E I and III


Overwinter, in paragraph one, most likely means:

A to hibernate
B to camouflage oneself
C to metamorphose
D to lower one's body temperature
E to avoid the cold


According to the passage above the life cycle of most ichneumons is still largely undocumented because:

A There has been little interest in the species.
B The bulk of their activity is concealed in inaccessible locations.
C Their tendency to sting makes close-range observation perilous.
D They only exist in geographically remote regions.
E Most ichneumons are so small as to be imperceptible to the naked eye.





# 4

Health scientists have long been puzzled by the so-called French Paradox -- the perplexing fact that the French consume at least as much fat and cholesterol as Americans do, smoke more cigarettes, and yet have far lower rates of coronary heart disease. After a thorough consideration of the two peoples' lifestyles, researchers have isolated yet another distinction between the French and the Americans. The French drink much more wine than do Americans, typically consuming it at meals.

These facts, combined with numerous studies conducted over the past twenty years, point to the fact that light to moderate consumption of alcohol, especially red wine, reduces the rate of heart disease dramatically. Research shows that men and women who drink a glass of wine a day exhibit a 20 to 50 percent lower risk of heart disease, and recent studies suggest that tempered consumption of alcohol reduces the risk of stroke.

Alcohol increases concentrations of HDL, the 'good' cholesterol that lowers the likelihood of heart disease and decreases platelet aggregability: it makes the blood less sticky and less likely to clot, thus decreasing the risk of a heart attack. With its many antioxidant components, including tannins, phenols, resveratrol, and quercitin, in addition to alcohol, red wine appears to be especially protective.


Based on the second paragraph of the passage, we can infer which of the following:

A Red wine is less expensive in France than in America
B More French vineyards produce red wine than white
C The positive effects of wine consumption compound the effects of cigarette smoking
D Of all alcoholic beverages, only red wine has beneficial medicinal effects
E Americans would be healthier if they drank more red wine


The word 'aggregability' in the third paragraph most likely means:

A the tendency to coagulate
B the tendency to oxidize
C the tendency to flatten
D the tendency to protect
E the tendency to dissipate


Based on the third paragraph, we can infer a connection between which of the following elements:

A 'good' cholesterol and 'bad' cholesterol
B oxidation and platelet aggregability
C 'good' cholesterol and blood clotting
D antioxidants and protection from heart disease
E resveratrol and HDL

The first sentence of the passage assumes that:

A The French have a healthier lifestyle than Americans
B There is a link between fat and cholesterol consumption and coronary heart disease
C Cigarette smoking is fashionable in France
D Coronary heart disease is usually fatal
E Levels of second-hand smoke in the air impact rates of coronary heart disease


According to this passage, the key to solving the French Paradox lies in understanding which of the following:

A The relationship between alcohol and HDL
B The relationship between cheese and 'bad cholesterol'
C The relationship between heart disease and stroke
D The relationship between alcohol and nicotine
E The relationship between environmental and genetic factors



# 5

The following text is adapted from the introduction to an 1871 book about Japan written by a British diplomat.

The books which have been written recently about Japan have either been compiled from official records, or have contained the sketchy impressions of passing travelers. Of the inner life of the Japanese the world at large knows but little: their religion, their superstitions, their ways of thought, the hidden springs by which they move—all these are as yet mysteries. Nor is this to be wondered at. The first Western men who came in contact with Japan—I am speaking not of the old Dutch and Portuguese traders and priests, but of the diplomats and merchants of eleven years ago—met with a cold reception. Above all things, the native Government threw obstacles in the way of any inquiry into their language, literature, and history. The fact was that the Tycoon's Government—with whom alone, so long as the Mikado remained in seclusion in his sacred capital at Kyoto, any relations were maintained—knew that the Imperial purple with which they sought to invest their chief must quickly fade before the strong sunlight which would be brought upon it so soon as there should be European linguists capable of examining their books and records. No opportunity was lost of throwing dust in the eyes of the newcomers, whom, even in the most trifling details, it was the official policy to lead astray. Now, however, there is no cause for concealment; the King has shaken off his sloth, and an intelligible Government, which need not fear scrutiny from abroad, is the result: the records of the country being but so many proofs of the Mikado's title to power, there is no reason for keeping up any show of mystery. The path of inquiry is open to all; and although there is yet much to be learnt, some knowledge has been attained, in which it may interest those who stay at home to share.

The recent revolution in Japan has wrought changes social as well as political; and it may be that when, in addition to the advance which has already been made, railways and telegraphs shall have connected the principal points of the Land of Sunrise, the old Japanese, such as he was and had been for centuries when we found him eleven short years ago, will have become extinct. It has appeared to me that no better means could be chosen of preserving a record of a curious and fast disappearing civilization than the translation of some of the most interesting national legends and histories, together with other specimens of literature bearing upon the same subject. Thus the Japanese may tell their own tale, their translator only adding here and there a few words of heading or tag to a chapter, where an explanation or amplification may seem necessary. I fear that the long and hard names will often make my tales tedious reading, but I believe that those who will bear with the difficulty will learn more of the character of the Japanese people than by skimming over descriptions of travel and adventure, however brilliant. The lord and his retainer, the warrior and the priest, the humble artisan and the despised Eta or pariah, each in his turn will become a leading character in my budget of stories; and it is out of the mouths of these personages that I hope to show forth a tolerably complete picture of Japanese society.

Having said so much by way of preface, I beg my readers to fancy themselves wafted away to the shores of the Bay of Yedo—a fair, smiling landscape: gentle slopes, crested by a dark fringe of pines and firs, lead down to the sea; the quaint eaves of many a temple and holy shrine peep out here and there from the groves; the bay itself is studded with picturesque fisher-craft, the torches of which shine by night like glow-worms among the outlying forts; far away to the west loom the goblin-haunted heights of Oyama, and beyond the twin hills of the Hakone Pass --Fujiyama, the Peerless Mountain, solitary and grand, stands in the centre of the plain, from which it sprang vomiting flames twenty-one centuries ago. For a hundred and sixty years the huge mountain has been at peace, but the frequent earthquakes still tell of hidden fires, and none can say when the red-hot stones and ashes may once more fall like rain over five provinces.

In the midst of a nest of venerable trees in Takanawa, a suburb of Yedo, is hidden Sengakuji, or the Spring-hill Temple, renowned throughout the length and breadth of the land for its cemetery, which contains the graves of the Forty-seven. Ronins, famous in Japanese history, heroes of Japanese drama, the tale of whose deeds I am about to transcribe.

And now for the story
.
In Japan, the decade before the writing of this text had been:

A A period of relative political stability.
B A period interrupted by volcanic eruptions.
C A time of social upheaval and political revolution.
D A time when the traditional Japanese roles of warrior, priest, and artisan were being rediscovered.
E A period of unprecedented production of new stories and legends regarding Japanese culture.


The phrase "no opportunity was lost of throwing dust in the eyes of newcomers" describes

A An effort to dazzle visitors with details of the literary and artistic accomplishments of the Japanese.
B The effects of ashes from Mt. Fujiyama.
C The efforts to keep holy shrines private.
D A custom observed since the arrival of Dutch and Portuguese traders.
E The previous government's efforts to deceive foreigners about its legitimacy.


What will follow in the remainder of the book?

A An account of the social changes that had recently occurred in Japan after the revolution.
B An analysis of the contents of the official government records that had previously been supressed.
C A novel by the author depicting the way of life of contemporary Japanese.
D A translation of various stories and legends of traditional Japanese culture.
E A description of the physical features of the Japanese landscape.


The author expects that over the next few years:

A Japan will revert to more traditional ways.
B The Ronins will resume leadership in Japan.
C The ancient class system will be reinstated.
D Modernization will likely transform the traditional social structure.
E International influences in Japan will diminish.


At the time the author wrote, Mount Fujiyama had been:

A A recently active volcano with violent eruptions comparable to the social revolution discussed in the text.
B Dormant for twenty one centuries.
C Dormant for the last 160 years.
D The solitary place of the seclusion of the Mikado.
E Discovered to be only one of a series of 47 other volcanoes.




The politically minded among us may do well to take a cue from the literati in our midst, our colleagues in the fields of language and literature who have contrived to sift through the layers of meaning inherent in language itself, unmasking the power of the metaphor as unpacking the associations of the symbol. As Americans, we take as given the decision made by the United States Congress on June 20, 1798 to adopt the bald eagle as our national emblem. And yet, though it graces our national currency and adorns our federal buildings, even the most reflective of us gives the bird scant thought. We are affronted by the notion of hunting it; we are concerned, if abstractly, that its numbers in the wild continue to decrease. But we pay little, if any mind, to the emblem itself, to the very sign and what it signifies.

Benjamin Franklin, himself a paragon of American virtue, opposed the bird known in his day as the American eagle as the choice for the national emblem. While the logic underlying his objection remains unarticulated, that of American ornithologist Arthur Cleveland Bent, who lived a century after Franklin, was unequivocal. He lamented that the bird's carrion-eating habits and its piratical attacks on the smaller, weaker osprey - itself a bird of prey - "hardly inspire respect and certainly do not exemplify the best in American character." Except, of course, that we have come, as a nation, to uphold the mercenary and the combative. Further fitting in these dubious times, is the fact that our beau idéal not only eats carrion, which it frequently steals from the osprey, but is also a skilled hunter, able to swoop down to capture water fowl in flight and rabbits on the run.

It is the eagle's competence as a hunter, its ability to secure food at will, that allows our eagle to perch for hours, conserving strength where other, inept birds squander their energy searching for food. So, to the charges of viciousness and thievery, we might add indolence: the bald eagle is a sedentary creature that often remains on the same branch or crag for long stretches of time. If we have begun to doubt the prudence, or perhaps more accurately, the desirability of emulating our national bird - of being a nation upholding alternately the virtues of violence, opportunism and laziness - let us conclude with one last comment about the behavior of the bird. When the eggs of the bald eagle hatch, the largest and strongest chick is very aggressive and frequently kills the younger and weaker birds in what is referred to as 'the Cain and Abel battle.' In this nation where the rich continue to prosper as the poor grow poorer, where the strong thrive and the weak languish, it would seem we have begun to perfect an imitative battle.

With which of the following statements would the author of this passage be least likely to agree?

A Americans can be mercenary and opportunistic.
B Americans can be violent.
C The American ideal of equality for all has been realized.
D The flaws of the bald eagle may be evidenced in the behavior of the American population.
E Americans tend toward laziness.


'Carrion' (paragraph 2) most likely means:

A a small bird of the warbler family
B a natural predator
C a flesh-eating animal
D the decaying flesh of a dead animal
E the flesh of caribou and reindeer


When the author states that Arthur Cleveland Bent was "unequivocal", he means to say that Bent was:

A condemning the symbol
B trying to provoke his listeners
C unwilling to compromise
D intending to spark controversy
E clear about his meaning


The author mentions all of the following as negative traits of the Bald Eagle except:

A gluttony
B indolence
C bloodthirst
D thievery
E savagery


The author's primary argument is that:

A The United States needs to select a new bird as its emblem.
B The values of the United States, as epitomized in the bald eagle, can scarcely be considered virtues.
C It is impossible to over-read the meaning of a symbol.
D Poverty in the United States threatens to undermine the quality of American life.
E The inevitable extinction of the bald eagle should not be lamented.




Paul Cezanne (1839-1906) was in many respects the archetypal artist, the tortured soul racked with doubt, filled with conflict. The doubt that engulfed him arose precisely because of the artistic conflict storming within him: in every stroke of the brush he waged the eternal struggle between chaos and order. He debated whether, like Poussin before him, he should create an ordered, harmonious vision of the world. Or should he acknowledge the evidence the world had set before him, that it inherently lacked order, that order was imposed - arbitrarily, even - from without? Could he acknowledge that the order of the world is entirely imposed by its observer, that we as human readers of the world around us discern patterns and impose them upon nature, that we construct meaningful frameworks to link together disparate components? Like the little child who lies in bed, closing one eye and then another, observing with each blink a shift in the room before her, Cezanne became more and more struck by the relativity of vision. It became increasingly significant to him that a slight tilt to the head, a closing of the eye, would alter the appearance of reality.

Cezanne longed to counter the fragmentariness of perceived reality; he sought, by contrast, precision and synthesis. Torn between a desire to represent the world as it is - the room seen with one eye closed and the world then seen with the other shut - and a desire to create a unified representation of the world, Cezanne ultimately arrived at an artistic compromise: the dialectic. He began to paint the variations that the slightly changed viewpoint allowed him to perceive. Rather than dedicate the one tree before him to the canvas, he would paint several possible trees. In his later work, he furthered his means of representing possibility, or uncertainty, by leaving a large portion of his canvas blank. One effect of this innovation was that it encouraged the viewer to envision her own variations of the variations already presented, thereby adding her own perspective to Cezanne's. Nature was no longer laid out before the viewer to be subjected to her scrutiny, rather it includes her, and the evidence of her own senses as part of the constantly changing world. With Cezanne, the viewer becomes part of the view. The consequence of the innovation was extremely significant for Cezanne himself, a first step toward taming the battle within him. By presenting the possibilities, he was able to establish certainty, a certainty paradoxically built on the acceptance of doubt.

Dialectic in paragraph 2 means:

A a reconciliation of opposites
B a logical argumentation
C a reasoned antithesis
D a study over a period of time
E a clear distinction


The author mentions Poussin in the first paragraph primarily in order to:

A Show the artistic tradition out of which Cezanne came.
B Give an example of a painter who influenced Cezanne.
C Offer an example of how radical Cezanne's move away from his predecessors was.
D Denote a painter who sought to order the world in his works.
E Indicate a painter aware of the impossibility of representing reality.


The author understands Cezanne's use of blank space to be:

A A brilliant innovation because it serves to include the viewer in the act of composition.
B A definitive break that allowed Cezanne to free himself from Poussin's influence.
C A necessary step that allowed Cezanne to cure himself of his neuroses.
D A reflection of Cezanne's laziness as a painter.
E An answer to the postmodern questions of fragmentariness and indeterminacy.


The tone of the author's discussion of Cezanne is best described as:

A cautiously enthusiastic
B unconditionally supportive
C blindly laudatory
D unduly deferential
E profoundly respectful


The author's main purpose in this passage is to:

A Argue that the art world would become stale without innovations like Cezanne's.
B Illustrate how childlike whimsy can produce artistic masterpieces.
C Critique those artists, who, unlike Cezanne, sought to order an inherently chaotic world.
D Detail the most significant contribution Cezanne made to art.
E Situate Cezanne among other famous artists.



The young man shut the door with a sharper slam than any visitor had used that afternoon, and walked up the street at a great pace, cutting the air with his walking-stick. He was glad to find himself outside that drawing-room, breathing raw fog, and in contact with unpolished people who only wanted their share of the pavement allowed them. He thought that if he had had Mr. or Mrs. or Miss Hilbery out here he would have made them, somehow, feel his superiority, for he was chafed by the memory of halting awkward sentences which had failed to give even the young woman with the sad, but inwardly ironical eyes a hint of his force. He tried to recall the actual words of his little outburst, and unconsciously supplemented them by so many words of greater expressiveness that the irritation of his failure was somewhat assuaged. Sudden stabs of the unmitigated truth assailed him now and then, for he was not inclined by nature to take a rosy view of his conduct, but what with the beat of his foot upon the pavement, and the glimpse which half-drawn curtains offered him of kitchens, dining- rooms, and drawing-rooms, illustrating with mute power different scenes from different lives, his own experience lost its sharpness.

His own experience underwent a curious change. His speed slackened, his head sank a little towards his breast, and the lamplight shone now and again upon a face grown strangely tranquil. His thought was so absorbing that when it became necessary to verify the name of a street, he looked at it for a time before he read it; when he came to a crossing, he seemed to have to reassure himself by two or three taps, such as a blind man gives, upon the curb; and, reaching the Underground station, he blinked in the bright circle of light, glanced at his watch, decided that he might still indulge himself in darkness, and walked straight on.

And yet the thought was the thought with which he had started. He was still thinking about the people in the house which he had left; but instead of remembering, with whatever accuracy he could, their looks and sayings, he had consciously taken leave of the literal truth. A turn of the street, a firelit room, something monumental in the procession of the lamp-posts, who shall say what accident of light or shape had suddenly changed the prospect within his mind, and led him to murmur aloud:

"She'll do. . . . Yes, Katharine Hilbery'll do. . . . I'll take Katharine Hilbery."

As soon as he had said this, his pace slackened, his head fell, his eyes became fixed. The desire to justify himself, which had been so urgent, ceased to torment him, and, as if released from constraint, so that they worked without friction or bidding, his faculties leapt forward and fixed, as a matter of course, upon the form of Katharine Hilbery. It was marvelous how much they found to feed upon, considering the destructive nature of Denham's criticism in her presence. The charm, which he had tried to disown, when under the effect of it, the beauty, the character, the aloofness, which he had been determined not to feel, now possessed him wholly; and when, as happened by the nature of things, he had exhausted his memory, he went on with his imagination. He was conscious of what he was about, for in thus dwelling upon Miss Hilbery's qualities, he showed a kind of method, as if he required this vision of her for a particular purpose. He increased her height, he darkened her hair; but physically there was not much to change in her. His most daring liberty was taken with her mind, which, for reasons of his own, he desired to be exalted and infallible, and of such independence that it was only in the case of Ralph Denham that it swerved from its high, swift flight, but where he was concerned, though fastidious at first, she finally swooped from her eminence to crown him with her approval. These delicious details, however, were to be worked out in all their ramifications at his leisure; the main point was that Katharine Hilbery would do; she would do for weeks, perhaps for months. In taking her he had provided himself with something the lack of which had left a bare place in his mind for a considerable time. He gave a sigh of satisfaction; his consciousness of his actual position somewhere in the neighborhood of Knightsbridge returned to him, and he was soon speeding in the train towards Highgate.

Based on the information provided in the first paragraph, which of the following best describes the feelings the young man is experiencing as a result of the meeting he just had with the Hilberrys?

A mild displeasure
B warm approval
C temporary indecision
D abject humiliation
E good-humored embarrassment


In the final paragraph, the young man's thoughts are mainly on:

A becoming more oriented to his physical surroundings
B reviewing the events of the earlier interview
C altering a previous impression of Hilberry
D remembering long forgotten aspects of Hilberry
E preparing a rebuttal to Denham on the subject of Hilberry


Which of the following best describes the transformation in the emotional state of the subject of this passage as he walks through the city?

A happiness to confusion
B indecision to abject sadness
C confused embarrassment to extreme anger
D agitation to resolve
E mild indignation to extreme anger


In the last paragraph the author writes that the young man’s faculties became fixed "without friction or bidding" on Katherine Hilberry. Which of the following best restates the author's meaning:

A the man’s thoughts unconsciously turned to Katherine Hilberry
B the man suddenly saw Katherine Hilberry in the lamplight
C the man’s thoughts towards Katherine Hilberry became less antagonistic
D the man stealthily approached Katherine Hilberry
E the man began replaying a pleasant conversation with Hilberry in his head


The word "chafed" as used in the first paragraph most nearly means:

A annoyed
B scraped
C made impatient
D warmed
E reminded





But man is not destined to vanish. He can be killed, but he cannot be destroyed, because his soul is deathless and his spirit is irrepressible. Therefore, though the situation seems dark in the context of the confrontation between the superpowers, the silver lining is provided by amazing phenomenon that the very nations which have spent incalculable resources and energy for the production of deadly weapons are desperately trying to find out how they might never be used. They threaten each other, intimidate each other and go to the brink, but before the total hour arrives they withdraw from the brink.

1. The main point from the author's view is that
1. Man's soul and spirit can not be destroyed by superpowers.
2. Man's destiny is not fully clear or visible.
3. Man's soul and spirit are immortal.
4. Man's safety is assured by the delicate balance of power in terms of nuclear weapons.
5. Human society will survive despite the serious threat of total annihilation.


2. The phrase 'Go to the brink' in the passage means
1. Retreating from extreme danger.
2. Declare war on each other.
3. Advancing to the stage of war but not engaging in it.
4. Negotiate for peace.
5. Commit suicide.

3. In the author's opinion
1. Huge stockpiles of destructive weapons have so far saved mankind from a catastrophe.
2. Superpowers have at last realized the need for abandoning the production of lethal weapons.
3. Mankind is heading towards complete destruction.
4. Nations in possession of huge stockpiles of lethal weapons are trying hard to avoid actual conflict.
5. There is a Silverlining over the production of deadly weapons.


4. 'Irrepressible' in the second line means
1. incompatible
2. strong
3. oppressive
4. unrestrainable
5. unspirited


5. A suitable title for the above passage is
1. Destruction of mankind is in evitable.
2. Man's desire to survive inhibits use of deadly weapons.
3. Mounting cost of modern weapons.
4. Threats and intimidation between super powers.
5. Cowardly retreat by man





Disequilibrium at the interface of water and air is a factor on which the transfer of heat and water vapor from the ocean to the air depends. The air within about a millimeter of the water is almost saturated with water vapor and the temperature of the air is close to that of the surface water. Irrespective of how small these differences might be, they are crucial, and the disequilibrium is maintained by air near the surface mixing with air higher up, which is typically appreciably cooler and lower in water vapor content. The turbulence, which takes its energy from the wind mixes the air. As the speed of wind increases, so does the turbulence, and consequently the rate of heat and moisture transfer. We can arrive at a detailed understanding of this phenomenon after further study. The transfer of momentum from wind to water, which occurs when waves are formed is an interacting-and complicated phenomenon. When waves are made by the wind, it transfers important amounts of energy-energy, which is consequently not available for the production of turbulence.

1. This passage principally intends to:
1. resolve a controversy
2. attempt a description of a phenomenon
3. sketch a theory
4. reinforce certain research findings
5. tabulate various observations


2. The wind over the ocean usually does which of the following according to the given passage?
I. Leads to cool, dry air coming in proximity with the ocean surface.
II. Maintains a steady rate of heat and moisture transfer between the ocean and the air.
III. Results in frequent changes in the ocean surface temperature.
1. I only
2. II only
3. I and II only
4. II and III only
5. I, II, and III


3. According to the author the present knowledge regarding heat and moisture transfer from the ocean to air as
1. revolutionary
2. inconsequential
3. outdated
4. derivative
5. incomplete


4. According to the given passage, in case the wind was to decrease until there was no wind at all, which of the following would occur?
1. The air, which is closest to the ocean surface would get saturated with water vapor.
2. The water would be cooler than the air closest to the ocean surface.
3. There would be a decrease in the amount of moisture in the air closest to the ocean surface.
4. There would be an increase in the rate of heat and moisture transfer.
5. The temperature of the air closest to the ocean and that of the air higher up would be the same.





The Food and Drug Administration has formulated certain severe restrictions regarding the use of antibiotics, which are used to promote the health and growth of meat animals. Though the different types of medicines mixed with the fodder of the animals kills many microorganisms, it also encourages the appearance of bacterial strains, which are resistant to anti-infective drugs.

It has already been observed that penicillin and the tetracyclines are not as effective therapeutically as they once used to be. This resistance to drugs is chiefly caused due to tiny circlets of genes, called plasmids, which are transferable between different species of bacteria. These plasmids are also one of the two kinds of vehicles on which molecular biologists depend on while performing gene transplant experiments. Existing guidelines also forbid the use of plasmids, which bear genes for resistance to antibiotics, in the laboratories. Though congressional dabate goes on as to whether these restrictions need to be toughened with reference to scientists in their laboratories, almost no congressional attention is being paid to an ill advised agricultural practice, which produces deleterious effects.

1. In the present passage, the author's primary concern is with:
1. The discovery of methods, which eliminate harmful microorganisms without generating drug-resistant bacteria.
2. Attempting an explanation of the reasons for congressional inaction about the regulation of gene transplant experiments.
3. Portraying a problematic agricultural practice and its serious genetic consequences
4. The verification of the therapeutic ineffectiveness of anti-infective drugs
5. Evaluation of the recently proposed restrictions, which are intended to promote the growth of meat animals.


2. As inferred from the above passage, the mutual transfer of plasmids between different bacteria can result in which of the following?
1. Microorganisms, which have an in-built resistance to drugs
2. Therapeutically useful circlets of genes
3. Penicillin like anti-infective drugs
4. Viruses used by molecular biologists
5. Carriers for performing gene transplant experiments.

3. According to the above passage the author believes that those who favor the stiffening of restrictions on gene transplant research should logically also.
1. Approve and aid experiments with any plasmids except those, which bear genes for antibiotic resistance.
2. Inquire regarding the addition of anti-infective drugs to livestock feeds
3. Oppose the using of penicillin and tetracyclines in order to kill microorganisms
4. Agree to the development of meatier live-stock through the use of antibiotics
5. Approve of congressional debate and discussion regarding science and health issues.


4. The attitude the author has with reference to the development of bacterial strains that render antibiotic drugs in effective can best be described as
1. indifferent
2. perplexed
3. pretentious
4. insincere
5. apprehensive




Roger Rosenblatt's book Black Fiction, manages to alter the approach taken in many previous studies by making an attempt to apply literary rather than sociopolitical criteria to its subject. Rosenblatt points out that criticism of Black writing has very often served as a pretext for an expounding on Black history. The recent work of Addison Gayle's passes a judgement on the value of Black fiction by clearly political standards, rating each work according to the ideas of Black identity, which it propounds.

Though fiction results from political circumstances, its author react not in ideological ways to those circumstances, and talking about novels and stories primarily as instruments of ideology circumvents much of the fictional enterprise. Affinities and connections are revealed in the works of Black fiction in Rosenblatt's literary analysis; these affinities and connections have been overlooked and ignored by solely political studies.

The writing of acceptable criticism of Black fiction, however, presumes giving satisfactory answers to a quite a few questions. The most important of all, is there a sufficient reason, apart from the racial identity of the authors, for the grouping together of Black authors? Secondly, what is the distinction of Black fiction from other modern fiction with which it is largely contemporaneous? In the work Rosenblatt demonstrates that Black fiction is a distinct body of writing, which has an identifiable, coherent literary tradition. He highlights recurring concerns and designs, which are independent of chronology in Black fiction written over the past eighty years. These concerns and designs are thematic, and they come form the central fact of the predominant white culture, where the Black characters in the novel are situated irrespective of whether they attempt to conform to that culture or they rebel against it.

Rosenblatt's work does leave certain aesthetic questions open. His thematic analysis allows considerable objectivity; he even clearly states that he does not intend to judge the merit of the various works yet his reluctance seems misplaced, especially since an attempt to appraise might have led to interesting results. For example, certain novels have an appearance of structural diffusion. Is this a defeat, or are the authors working out of, or attempting to forge, a different kind of aesthetic? Apart from this, the style of certain Black novels, like Jean Toomer's Cane, verges on expressionism or surrealism; does this technique provide a counterpoint to the prevalent theme that portrays the fate against which Black heroes are pitted, a theme usually conveyed by more naturalistic modes of expressions?

Irrespective of such omissions, what Rosenblatt talks about in his work makes for an astute and worthwhile study. His book very effectively surveys a variety of novels, highlighting certain fascinating and little-known works like James Weldon Johnson's Autobiography of an Ex-Coloured Man. Black Fiction is tightly constructed, and levelheaded and penetrating criticism is exemplified in its forthright and lucid style.

1. The author of the passage raises and objection to criticism of Black fiction like that by Addison Gayle as it:
1. Highlights only the purely literary aspects of such works
2. Misconceive the ideological content of such fiction
3. Miscalculate the notions of Black identity presented in such fiction
4. Replaces political for literary criteria in evaluating such fiction
5. Disregards the reciprocation between Black history and Black identity exhibited in such fiction.


2. The primary concern of the author in the above passage is:
1. Reviewing the validity of a work of criticism
2. Comparing various critical approaches to a subject
3. Talking of the limitations of a particular kind of criticism
4. Recapitulation of the major points in a work of criticism
5. Illustrating the theoretical background of a certain kind of criticism.


3. The author is of the opinion that Black Fiction would have been improved had Rosenblatt:
1. Undertaken a more careful evaluation of the ideological and historical aspects of Black Fiction
2. Been more objective in his approach to novels and stories by Black authors
3. Attempted a more detailed exploration of the recurring themes in Black fiction throughout its history
4. Established a basis for placing Black fiction within its own unique literary tradition
5. Calculated the relative literary merit of the novels he analyzed thematically.


4. Rosenblatt's discussion of Black Fiction is :
1. Pedantic and contentious
2. Critical but admiring
3. Ironic and deprecating
4. Argumentative but unfocused
5. Stilted and insincere.


5. According to the given passage the author would be LEAST likely to approve of which among the following?
1. Analyzing the influence of political events on the personal ideology of Black writers
2. Attempting a critical study, which applies sociopolitical criteria to the autobiographies of Black authors
3. A literary study of Black poetry that appraises the merits of poems according to the political acceptability of their themes
4. Studying the growth of a distinct Black literary tradition within the context of Black history
5. Undertaking a literary study, which attempts to isolate aesthetic qualities unique to Black fiction.


6. From the following options, which does the author not make use of while discussing Black Fiction?
1. Rhetorical questions
2. Specific examples
3. Comparison and contrast
4. Definition of terms
5. Personal opinion.


7. The author makes a reference to James Weldon Johnson's Autobiography of an Ex-colored Man most probably to:
1. Highlight the affinities between Rosenblatt's method of thematic analysis and earlier criticism
2. Elucidate regarding the point made regarding expressionistic style earlier in the passage
3. Qualify the assessment of Rosenblatt's book made in the first paragraph of the passage
4. Demonstrate the affinities among the various Black novels talked of by Rosenblatt's literary analysis
5. Present a specific example of one of the accomplishments of Rosenblatt's work.




Some modern anthropologists hold that biological evolution has shaped not only human morphology but also human behavior. The role those anthropologists ascribe to evolution is not of dictating the details of human behavior but one of imposing constraints - ways of feeling, thinking, and acting that ''come naturally'' in archetypal situations in any culture. Our ''frailties'' - emotions and motivs such as rage, fear, greed, gluttony, joy,lust, love-may be a very mixed assortment quality: we are, as we say, ''in the grip'' of them. And thus they give us oursense of constraints.

Unhappily, some of those frailties our need for ever-increasing security among them are presently maladaptive. Yet beneath the overlay of cultural detail, they, too, are said to be biological in direction, and therefore as natural to us as are our appendixes. We would need to comprehend throughly their adaptive origins in order to understand how badly they guide us now. And we might then begin to resist their pressure.

1. The author implies that control to any extent over the ''frailties'' that constrain our behavior is though to presuppose
1. That those frailties and adaptive are recognized as currently beneficial and adaptive
2. That there is little or no overlay of cultural detail that masks their true nature.
3. That there are cultures in which those frailties do not ''come naturally'' and from which such control can be learned
4. A full understanding of why those frailties evolved and of how they function now
5. A thorough grasp of the principle that cultural detail in human behavior can differ arbitrarily from society to society.


2. It can be inferred that in his discussion of maladaptive frailties the author assumes that
1. Evolution does not favor the emergence of adaptive characteristics over the emergence of maladaptive ones
2. Any structure or behavior not positively adaptive is regarded as transitory in evolutionary theory
3. Maladaptive characteristics, once fixed, make the emergence of other maladaptive characteristics more likely
4. The designation of a characteristic as being maladaptive must always remain highly tentative
5. Changes in the total human environment can outpace evolutionary change.

3. The primary purpose of the passage is to present
1. A position on the foundations of human behavior and on what those foundations imply
2. A theory outlining the parallel development of human morphology and of human behavior
3. A diagnostic test for separating biologically determined behavior patters from culture - specific detail
4. An overview of those human emotions and motive's that impose constraints on human behaviour
5. A practical method for resting the pressures of biologically determined drives.


4. Which of the following most probably provides an appropriate analogy from human morphology for the ''details'' versus ''constraints'' distinction made in the passage in relation to human behaviour?
1. The ability of most people to see all the colors of the visible spectrum as against most peoples inability to name any but the primary colors
2. The ability of even the least fortunate people to show compassion as against people's inability to mask their feelings completely
3. The ability of some people to dive to great depths as against most people's inability to swim long distance
4. The psychological profile of those people who are able to delay gratification as against people's inability to control their lives completely
5. The greater lung capacity of mountain peoples that helps them live in oxygen-poor air as against people's inability to fly without special apparatus.

Friday, August 29, 2008

29 August Gre Verbal Question

1.Gruff

2.Rivet

3.Pacify

4.Qurush

5. Repudiate

6. Complaisance

7. Encomium

8. Embezzle

9. Adamant

10. Demoralized

11. Natatorium

12. Imbue

13. Vapid

14. Clairvoyant

15. Obscene




ANALOGIES:


1.Game: Stadium

2.Brake: speed

3.Embezzling: stealing

4.Generator: Electricity

5.Factory: Product

29th august GRE Quants Question

1. If a/b @ c = (a/b) / c then (7/3) @ 4 =?

2. Col A: 100! / 99!
Col B: (100! – 99!) / 98!

3. Given three points a, b, c. If a, b lie on the same line and if all these three points are at same distance between each other, then
A. None of the points lie on the same plane.
B. Only one point lies on the plane.
C. Two points lie on the plane.
D. More than two points lie on the plane.
E. None of the above.

4. Given a series -9, 10, -11, 12, -13. . . . . . . . . . .80
Col A: sum of first 27 numbers
Col B: -22

5. If 100 < Y
Col A: 2y
Col B: 400

6.

As in above figure a, b and c are three angles & it is given that RQ is parallel to NO and PQ is parallel to MN.
Col A: a
Col B: b+c

Thursday, August 28, 2008

28 th August GRE Quats Question

1. If N is an odd and negative integer and it is a product of ten different integers.
Col A: no. of odd integers used in the product to get N
Col B: no. of negative integers used in the product to get N

2. Given a figure of combination of two triangles, in which first triangle contains 90, x, y degree angles and second triangle contains 90, z, w degrees. Which of the following conditions must be true?
I. x - w = z - y
II. x + y = w
III. z = 90 - w
A. I only
B. II only
C. I and III only
& so on....

3. Given a sequence -9, 10, -11, 12, -13, 14, -15..... Find the sum of first 40 integers in sequence?

4. If n > 1 then
Col A: (-1)^(4n-3)
Col B: (-1)^(4n-2)

5.

If all the seven circles are shaded like above then what is the area of unshaded region?

6. The standard deviation of the series 3, 7, 9, 13, 17 is?

7. Col A: 77!/78!
Col B: 87!/88!

8. If 2^n > 10^15n then which of the following options satisfies the condition
A. 30
B. 45
C. 60
D. 75

9. If x, y are the prime numbers greater than 10, then which of following must be even?
A. (x^2)*(y^2)
B. x^2 + x*y
C. 2*x*y + x
& so on....

10. Given a line in xy-plane with slope k = -3 & the points are (2, k), (-3, m)
Col A: k-m
Col B: -15

11. The approximate value of ((61.16)(.9Cool^2)/sqrt(401) is
A. 5
B. 4
C. 3
D. 2
E. 1

12. Col A: 0.9999/0.9998
Col B: 1.0002/1.0001

13. There is a junior and senior team who has taken tests. The average of junior team members who have taken tests are 88 and the average of senior team members is 92
Col A: the average of senior and junior team
Col B: 90

14. When a number is divided by 12, the remainder is 5. What is the remainder when the square of that number is divided by 8?

15. If the selling price of a product is 25% lesser than its list price and 40% greater than its cost price, then what is the list price of the product if the cost price = 30 ?

16. Last year, 1/4 of factory workers are architects. One year ago 60 workers newly joined in which 50 are architects and no one left the factory since last year and if now 1/3 of factory workers are architects then find the total number of workers?

17. When 'w' is divided by 14, the remainder is zero. If 'w' is three lesser than it value and when divided by 15, its remainder is 14. What is the value of 'w'?

18. If RST is isosceles triangle with RS = ST and P be a point on the side RT, then which of the following conditions must be true:
i) SP < ST
ii) SP < RT
iii)SP < PT

Monday, August 25, 2008

25th August Gre Verbal Question

Today's Words:


1. Exacerbate

2. Stern

3. Debauched

4. Lampoon

5. Impeachable

6. Gratuitous

7. Feral

8. Amenable

9. Futility

10. Pragmatic

11. Saboteur

12. Disrupt




Analogies:

1. Starryeyed: Pragmatic

2. Saboteur: Disrupt

3. choreograph:movement

25th August Gre Quants Question

If AB = BC, CI = IH & DE = EF then
Col A: Area of Rectangle GDEF
Col B: Sum of areas of Rectangle ABCD & CIHE

2. What is the closest integer to sqrt(171)?

3. If an = n { (-1)^n – 1)}, and a1, a2, a3, a4, a5 are the terms then what is the difference between greatest & least term?

4. No. of Members Least weight Range
20 members 100 29
21members 130 35
When two groups are joined, what is the median weight of the group?

5. Col A: 0.9999/0.9998
Col B: 1.0002/1.0001

6. If (2x+7) < 13 then
Col A: x^2
Col B: 9

7.

If AC = x, then find the perimeter of the figure?

Friday, August 22, 2008

Barrons Gre Flash Card

Just Take out the print out and fold the Card in appropriate manner so that u can hide the answer of gre word and check your answer by seeing back.

Read this document on Scribd: Barron FlashCard

22 august gre quats question

1. There is a junior and senior team who have taken tests. The average of junior team members who have taken tests are 88 and the average of senior team members is 92
Col A: the average of senior and junior team
Col B: 90

2. Given 0.61 Col A: square root(x)
Col B: 0.73

3. A toy manufturer company manufactures 20,000 toys and exports ¾ and 1/10 and donates some (xxx) value for charitable trust.
Find the number of toys left in the company?

4. There are rooms from 101 to 550 inclusive in which the starting rooms are numbered from 1, 2, 3 and the waiting rooms are numbered from 4,5,6. What is the probability that the rooms 4, 5, 6 will be choosed randomly?

5. One side of a triangle is given as 6 and the triangle is equilateral triangle & another triangle is joined with it whose side is 10
Col A: perimeter of polygon
Col B: 30
(Question is similar to this)

6. Given a line in xy plane with slope k = -1/2
Col A: x-intercept
Col B: y-intercept

7. If x < y then
Col A: x^2+x+1
Col B: y^2+y+1

8. Which of the following number is nearer to sqrt(171)?
A.12
B.13
C.14
D.15
E.16

9. When a number is divided by 12, the remainder is 5. What is the remainder when the square of that number is divided by 8?

10. The area of circle of radius 'r' is twice the the area of triangle then r in terms of h is?

11. Col A: 0.001^-1 + 0.999^-1
Col B: 0.002^-1 + 0.998^-1

12. If the selling price of a product is 25% lesser than its list price and 40% greater than its cost price, then what is the list price of the product if the cost price = 30 ?
A. 52
B. 53
C. 56
& so on.....

13. If x and y are prime numbers greater than 10 then which of the following is an even integer?
A. x^2+xy
B. x^y +2
C. xy +2
D. 2xy +xy

14. If z>1 & z(x+y+z) = zx+zy+z
Col A: z
Col B: 0

15. If x<2 & y<3
Col A: xy
Col B: 6

16. The average salary of n employees is 32000, average salary of another different group of n-1 employees is 33000
Col A: The average salary of 2n-1 employees
Col B: 32500

17. The range of 'p' numbers is 9.5 and the range for 'q' numbers is 21.5, what is the least range value when p and q are combined together?

18. A series was given as 1, -3, 5,-7 ,9,-11,13............ In this series odd numbers are negative integers. Find the sum of first 25 numbers in the series?

19. If n is an integer then
Col A: (-1)^6n+9
Col B: (-1)^2n-1

20. A trapezoid is given whose area is 12, length of it is given as 6 & height of it is given as 3. Find the perimeter of trapezoid?

Antonyms Test 2

ABRASION : STROKE ::

(A) boil : heat
(B) rubble : demolish
(C) flattery : compliment
(D) remnant : cut
(E) nut : shell


JUDGE : IMPARTIAL ::

(A) animal : tame
(B) acrobat : limber
(C) dignitary : proud
(D) prisoner : repentant
(E) politician : liberal


DRAWER : BUREAU ::

(A) tributary : river
(B) trunk : tree
(C) article : magazine
(D) kernel : husk
(E) language : dictionary


BATHE : WATER ::

(A) spread : knife
(B) fly : airplane
(C) obstruct : plug
(D) point : finger
(E) cover : coat


ILLICIT : RATIFY ::

(A) obsolete : preserve
(B) confusing : obscure
(C) popular : criticize
(D) insignificant : highlight
(E) belligerent : appease


PROTAGONIST : CHARACTER ::

(A) brush : applicator
(B) lawmaker : government
(C) costume : gala
(D) conductor : orchestra
(E) novice : competitor



CULTIVATE : GARDEN ::

(A) play : game
(B) worship : church
(C) explore : cave
(D) relax : vacation
(E) steer : automobile


EMPLOY : INDIGENCE ::

(A) expose : dormancy
(B) arrive : presence
(C) submerge : buoyancy
(D) season : blandness
(E) know : ignorance


SIEVE : DIVIDE ::

(A) staple : collate
(B) funeral : mourn
(C) siphon : squeeze
(D) panacea : remedy
(E) monarch : serve


MEDIATE : MEDDLE ::

(A) reason : think
(B) accumulate : hoard
(C) neglect : abandon
(D) surpass : cheat
(E) discourage : burde


1. MASON :: STONE

A. soldier : weapon
B. lawyer : law
C. blacksmith : forge
D. teacher : pupil
E. carpenter : wood

2. ARTICULATE :: SPEECH

A. predictable : event
B. coordinated : movement
C. dangerous : disease
D. active : thought
E. erratic : path

3. INCEPTION :: CONCLUSION

A. departure : arrival
B. culmination : upshot
C. refutation : approval
D. approach : return
E. escapade : punishment

4. SCINTILLATING :: DULLNESS

A. erudite : wisdom
B. desultory : error
C. boisterous : calm
D. cautious : restraint
E. exalted : elevation

5. ELUCIDATE :: CLARITY

A. envision : memory
B. aggravate : problem
C. conceal : oblivion
D. illuminate : light
E. mystify enlightenment

6. SHARD :: POTTERY

A. seed : flower
B. smoke : fire
C. dish : menu
D. chair : furniture
E. splinter : wood

7. REPEL :: LURE

A. dismount : devolve
B. abrogate : deny
C. abridge : shorten
D. enervate : weaken
E. miscarry : succeed

8. PENURY :: MONEY

A. starvation : sustenance
B. independence : freedom
C. infirmity : illness
D. reality : foresight
E. spontaneity : care

9. ARABLE :: LAND

A. impenetrable : jungle
B. navigable : waterway
C. fertile : fertilizer
D. shallow : pond
E. flat : field

10. ATTENUATE :: SIGNAL

A. exacerbate : problem
B. modify : accent
C. dampen : enthusiasm
D. elongate : line
E. dramatize : play

11. FURNACE :: SLAG

A. vegetable : garbage
B. factory : goods
C. fire : ashes
D. automobile : gasoline
E. silo : grain

12. PROSAIC :: MUNDANE

A. obdurate : foolish
B. ascetic : austere
C. clamorous : captive
D. loquacious : taciturn
E. peremptory : spontaneous

13. SALACIOUS :: WHOLESOME

A. religious : private
B. expensive : profligate
C. conservative : stoic
D. mendacious : truthful
E. fulsome : generous

14. PONDER :: PROBLEM

A. remove : doubt
B. capture : runaway
C. seize : time
D. ruminate : idea
E. curl : hair

15. HACKNEYED :: FRESHNESS

A. stale : porosity
B. facile : delicacy
C. ponderous : lightness
D. central : vitality
E. relevant : pertinence



1. ELECTED :: INAUGURATION

A. enrolled : graduation
B. condemned : execution
C. chosen : selection
D. gathered : exhibition
E. appointed : interview

2. DIVIDEND :: STOCKHOLDER

A. patent : inventor
B. royalty : author
C. wage : employer
D. interest : banker
E. investment : investor

3. ARCHIPELAGO :: ISLAND

A. village : hamlet
B. constellation : star
C. river : sea
D. finger : hand
E. tongue : mouth

4. PRECIS :: ELABORATION

A. comprehension : understanding
B. revision : correction
C. simplification : decoration
D. accuracy : abnormality
E. expurgation : distortion

5. MINCE :: WALK

A. bang : sound
B. wave : gesture
C. waltz : dance
D. simper : smile
E. hike : run

6. DISINTERESTED :: UNBIASED

A. indulgent : intolerant
B. exhausted : energetic
C. languid : lethargic
D. unconcerned : involved
E. profligate : flippant

7. AMBIVALENT :: CERTAIN

A. indifferent : biased
B. furtive : open
C. impecunious : voracious
D. discreet : careful
E. munificent : generous

8. DEFERENCE :: RESPECT

A. admiration : jealousy
B. condescension : hatred
C. affection : love
D. pretence : truth
E. gratitude : charity

9. AUTHORITARIAN :: LENIENT

A. philanthropist : generous
B. virtuoso : glamorous
C. hedonist : indulgent
D. servant : servile
E. miser : charitable

10. ALLAY :: SUSPICION

A. tend : plant
B. impede : anger
C. calm : fear
D. fell : tree
E. exacerbate : worry

11. PERENNIAL :: EPHEMERAL

A. volatile : evanescent
B. mature : ripe
C. diurnal : annual
D. permanent : temporary
E. majestic : mean

12. DIRECTORY :: ADDRESS

A. list : number
B. catalogue : shop
C. thesaurus : fact
D. dictionary : spelling
E. encyclopaedia : solecism

13. PRIMITIVE :: SOPHISTICATE

A. employee : superior
B. socialite : recluse
C. tyro : expert
D. native : inhabitant
E. applicant : member

14. TAP :: PUMMEL

A. tiptoe : stamp
B. hit : hurt
C. lumber : stumble
D. talk : chat
E. pump : water

15. TRAGEDY :: DRAMA

A. farce : actor
B. cartoon : film
C. prosody : poem
D. accident : ambulance
E. epigram : anecdote



BOAST :: LANGUAGE

A. glare : anger
B. swagger : gait
C. stare : sight
D. grow : height
E. enrage : anger

2. BELITTLE :: DISPARAGE

A. jeopardize : protect
B. efface : inscribe
C. assuage : increase
D. deride : ridicule
E. decrease : augment

3. VIRULENT :: INNOCUOUS

A. pretentious : harmful
B. reprehensible : praiseworthy
C. strong : delicate
D. epidemic : widespread
E. antiseptic : medical

4. INCOHERENT :: CLARITY

A. quiet : volume
B. normal : austerity
C. stagnant : light
D. scribbled : writing
E. tidy : mind

5. DESECRATE :: HOLY

A. despoil : beautiful
B. beautify : ugly
C. glamorize : rich
D. damage : corporeal
E. improve : dull

6. VIRTUOSO :: ACCOMPLISHED

A. aesthete : austere
B. servant : servile
C. hedonist : pretentious
D. priest : orthodox
E. philanthropist : generous

7. AUTHORITARIAN :: STRICT

A. fallacious : fraught
B. fastidious : particular
C. exemplary : arrogant
D. apprehensive : eager
E. neutral : objective

8. HOMOGENEOUS :: KIND

A. fast : speed
B. suspicious : origin
C. diverse : route
D. contemporary : time
E. disparate : place

9. CONSPICUOUS :: VISIBLE

A. sensual : audible
B. irrevocable : changed
C. elastic : stretched
D. ignominious : denounced
E. sensitive : felt

10. AUTHENTIC :: APOCRYPHAL

A. immune : dangerous
B. conventional : unorthodox
C. pious : religious
D. eccentric : strange
E. reticent : chaotic

11. TEMPER :: HARD

A. mitigate : severe
B. provoke : angry
C. endorse : tough
D. infer : certain
E. scrutinize : clear

12. STANZA :: POEM

A. chapter : novel
B. prose : verse
C. stave : music
D. song : chorus
E. overture : opera

13. ABSTEMIOUS :: INDULGE

A. subversive : undermine
B. diffident : confide
C. profligate : spend
D. gregarious : participate
E. submissive : assert

14. FRAGMENT :: MERGE

A. engage : marry
B. loose : tighten
C. splinter : join
D. mend : rend
E. diverge : convex

15. PROLIFIC :: UNFRUITFUL

A. new : old
B. blooming : withered
C. fertile : barren
D. fresh : stale
E. fecund : fervid

Antonyms Test -1

ZENITH

A apogee
B nadir
C collapse
D apotheosis
E denial

INGENUOUS

A naive
B insincere
C thrifty
D lachrymose
E innocuous

ATTENUATE

A mollify
B dilute
C enhance
D incapacitate
E distend

SYCOPHANT

A cynic
B critic
C philanderer
D philanthropist
E follower

RETICENCE

A candor
B silence
C denunciation
D powerlessness
E gentility

IMPERIOUS

A diffident
B gregarious
C egregious
D facetious
E overbearing

PROBITY

A rectitude
B infidelity
C corruptibility
D infamy
E uprightness


SANGUINE

A perturbed
B pedantic
C trite
D pessimistic
E defective


BUCOLIC

A pastoral
B urban
C idyllic
D chimerical
E chromatic


FRACTIOUS

A tacit
B brittle
C docile
D durable
E delicate

EFFULGENT

A concise
B drab
C vivacious
D terse
E sane


ABLUTION

A pollution
B lavation
C fumigation
D debasement
E decompositon


OBSTREPEROUS

A inchoate
B mundane
C compliant
D celebrated
E messy


SCURRILOUS

A fastidious
B scandalous
C decent
D voluptuous
E evocative

INVETERATE

A decrepit
B habitual
C chronic
D conditional
E novel

PICAYUNE

A grandiose
B extravagant
C trivial
D magnanimous
E immense


VALUE

A assess
B appreciate
C misestimate
D depreciate
E disdain


LETHARGY

A excitement
B torpor
C delirium
D drowsiness
E verve

SALUBRIOUS

A detrimental
B mendacious
C indignant
D incorruptible
E elevated

DILATORY

A serious
B prompt
C undefeated
D jovial
E indolent

INAUGURAL

A initial
B ultimate
C penultimate
D conterminous
E determinate


PERSPICACIOUS

A beautiful
B domineering
C unobservant
D dynamic
E persuasive


MERCURIAL

A staid
B capricious
C dedicated
D irascible
E simple


SWAY

A power
B control
C impotence
D insignificance
E illegitimacy


HALE

A infirm
B healthy
C cancerous
D injured
E vigorous


SOPORIFIC

A analgesic
B mature
C callous
D arousing
E emphatic


QUIXOTIC

A timid
B deleterious
C inoffensive
D pragmatic
E erratic

MAWKISH

A arrogant
B divisive
C creative
D indecisive
E unsentimental

PARSIMONIOUS

A feral
B cordial
C affable
D prodigal
E irascible

VOLUBLE

A glib
B infinitesimal
C taciturn
D boisterous
E frigid


SYBARITE

A hedonist
B accomplice
C protegee
D friend
E ascetic

PHLEGMATIC

A excitable
B dogmatic
C destructive
D ineluctable
E handsome

TACT

A imprudence
B misconduct
C indiscretion
D finesse
E thoughtlessness

DEFILE

A purify
B rectify
C infuse
D organize
E restore


SURREPTITIOUS

A clandestine
B overt
C covert
D stealthy
E rife

INTRANSIGENT

A passive
B complacent
C meek
D conciliatory
E forward

APPROBATION

A commendation
B liberation
C qualification
D alliance
E disapproval

SAGACITY

A illness
B convalescence
C vaccination
D inexperience
E ebullience

MALLEABLE

A docile
B intractable
C derogatory
D impressionable
E voracious


CAPACIOUS

A unfettered
B cramped
C deleterious
D callous
E egregious


APHASIA

pain
illness
solid state
speech loss
volubility


BESTIAL


noble
zoological
excellent
fuzzy
poor

ANTIPATHY

willingness
hatred
fondness
unwillingness
misery


ANTITHESIS

resurrection
destruction
opposite
similarity
creation

AMBULATORY

bedridden
evolutionary
emerging
emergency
congenital

BERATE

praise
accept
refer
deny
purchase


CHASTE

bold
wanton
hazy
followed
clear

CHIDE

unite
avoid
praise
help
follow

BIGOTRY

monogamy
arrogance
mourning
tolerance
polygam

BAROQUE

loud
simple
criticize
clumsy
monologue


CHURLISH


polite
religious
upset
rude
compressed


CAPRICIOUS


scattered
greedy
satisfied
steadfast
phenomenal


CARTE BLANCHE


restrictions
freedom
socialistic
commumal
capitalism

AMALGAMATE


conglomerate
separate
mixture
responsible
correspond

AMELIORATE


inflation
deflation
improve
recuperate
make worse

AMBIGUOUS


salvagable
hazy
clear
fuzzy
ancillary



BIZARRE


strange
normal
extol
staged
triumphant

CAPTIOUS


ambitious
capable
tolerant
humorous
incapable

ANONYMOUS


signed
expert
saint
unknown
subtle

ANTEDILUVIAN


ancient
ruins
old
modern
remains


# ABOMINATE :

1. loathe
2. despise
3. adore
4. abhor
5. attach


# OBSEQUIOUS :

1. servile
2. first
3. fawning
4. supercilious
5. improper


# OROTUND :

1. not resonant
2. not reddish
3. not eager
4. pompous
5. loud


# RECANT :

1. entangle
2. rescue
3. fail
4. assert
5. predict


# UPBRAID :

1. defer
2. vacillate
3. sever
4. conjoin
5. laud


# PLENITUDE :

1. luxury
2. magnificence
3. richness
4. contentment
5. scarcity


# SCURRILOUS :

1. decent
2. savage
3. major
4. volatile
5. scabby


# FULMINATION :

1. praise
2. repetition
3. escape
4. ratification
5. addition


# DISTEND

1. deteriorate
2. weaken
3. constrict
4. concentrate
5. fold


# TOUT

1. cast aspersions on
2. deny the relevance of
3. placate
4. withhold consent
5. misrepresent

# SQUALID

1. fervid
2. florid
3. pristine
4. extraneous
5. abundant


# SCOTCH

1. renovate
2. entrust
3. unfasten
4. encourage
5. emphasize


# PERFIDY

1. tact
2. generosity
3. thoroughness
4. loyalty
5. gratitude


# OUTLANDISH

1. conventional
2. prolific
3. unchanging
4. transparent
5. noticeable


# PLUMB

1. reversed
2. lofty
3. horizontal
4. thin
5. light


# FERVID

1. undistinguished
2. unexpected
3. stubborn
4. restrained
5. discouraged


# VACUITY

1. quality
2. certainty
3. plenitude
4. stability
5. incontinence


# RAVEL

1. knit
2. omit
3. remain silent
4. measure
5. increase in value


# PERSISTENCE

1. irrelevance
2. inconstancy
3. inequality
4. intemperance
5. incompetence.


# SUBROSA

1. openly
2. fashionably
3. under the owse
4. simply
5. clandestinely

General Strategies for Reading Comprehension

General Strategies for Reading Comprehension

1. Try to read the whole text of the passage once, if possible. Many people think you should just skim the passage or read the first lines of every paragraph, and not to read the passage. We believe this is an error: if you misunderstand the main idea of the passage, you will certainly get at least some of the questions wrong. Give the passage one good read, taking no more than 3 minutes to read all of the text. Do not read the passage more than once – that wastes too much time. If you have not understood it completely, try to answer the questions anyway. Note: this point of reading the whole passage is important for test-takers whose first language is not English, provided that they can read the passage in 3 minutes or less.

2. Make brief notes on the text on your scrap paper. As we will see below in greater detail, you should write down a couple of words on A) the Main Idea or Primary Purpose, B) Organization/Structure of the passage, and C) the Tone or Attitude of the author (if applicable). You just need a few words for each of these areas, and altogether it should not take longer than 30 seconds to write down.

3. Remember that the tone or attitude of the passage is usually respectful and moderate, never going to extremes of praise nor criticism. ETS obtains its Reading Comprehension passages from real articles about real academics and professionals. So the tone of the articles, even when there is criticism in the passage toward an academic or her work, is always balanced and moderate. In the same vein, articles that deal with minorities or ethnic groups are almost always positive and sympathetic.

4. Look out for structural words that tell you the important ideas or transitions in a passage.

Continue the Idea Words
Similarly
Moreover
Additionally
In the same way
Likewise

Conclusion Words
Thus
Therefore
Hence
So
In summary
In conclusion

Contradiction or Contrast Words
Neverthless
Nonetheless
However
But
Although
Though
Even though
Notwithstanding
Yet
Despite
In spite of
On the one hand…on the other hand
While
Unlike

5. Go back to the text of the passage for the answers. Many test-takers fail to return to the text of the passage to look for the correct answers. They rely solely on their memories and understanding of the passage after having read or skimmed it. Wrong. ETS is counting on that. Go back to the text to look for information to answer the questions. Nine times out of ten, the answer lies within the passage.

Of the 6 most important types of questions for Reading Comprehension, we will first look at Main Idea/Primary Purpose Questions, and the strategies we can use to answer them.
Main Idea/Primary Purpose Questions
Many people believe there is no difference between the main or central idea of the passage and the primary purpose of the author of the passage. This is simply not true. Let's take a look at the subtle but important difference between them:
Main Idea
The question might look something like this:

"Which of the following best states the central idea of the passage?"
"Which of the following most accurately states the main idea of the passage?"
"Which of the following is the principal topic of the passage?"
"The main topic of the passage is...."

Primary Purpose
The question might look like this:

"The primary purpose of this passage is to..."
"The primary purpose of the passage as a whole is to..."
"The primary focus of this passage is on which of the following?"
"The main concern of the passage is to..."
"In the passage, the author is primarily interested in...."
"The passage is chiefly concerned with..."
Strategy:
Main Idea: Look in the first and last paragraphs for the main idea. Any conclusion words like therefore, thus, so, hence, etc. that you see are most likely introducing the main idea. The correct answer will say the same thing as it says in the text, but using different words. The Main Idea is not always stated explicitly in the passage – in fact, more likely than not, it is not stated explicitly. Therefore, in order to answer this type of question when it is more implicit:

1. Re-read the first line of every passage, and the last line of the first and last paragraphs. This should give you the general structure or outline of the argument, with which you can answer the Main Idea question.
2. After determining the general structure or content of the argument, eliminate answer choices that are too broad or too specific, i.e. answer choices that go beyond the content of the passage, or that deal with content only discussed in one paragraph of the passage.
3. Make brief notes – a couple of words- regarding the Main Idea on the text on your scrap paper while you read.
Primary Purpose: What is the author trying to do? What is his intention? If he is evaluating a theory, then the answer could be something like "Discuss an interpretation". Note that the correct answer would deal with "an interpretation", because the author is only dealing with one theory. If the Primary Purpose is to criticize 2 new books, then his intention or his primary purpose might be to "Critique new studies". Again, as in Main Idea questions, re-read the first line of every passage, and the last line of the first and last paragraphs. This should give you the general structure or outline of the argument, with which you can answer the Primary Purpose question.
Note: A good main idea or primary purpose does not go beyond the scope of the passage, nor does it limit itself to discussing only one part of the passage.


What is the primary purpose of this passage?

A)
discuss the importance of the television program Star Trek for the international space program
B) discuss important theoretical work concerned with faster-than-light space travel.
C) explore a dispute among theoretical physicists regarding the uses of space flight
D) describe the possible uses of space-warping material
E) explain how a space-warping bubble would work in the real world

Explanation
This is a Primary Purpose question, so we have to determine what the author is trying to do or say in this passage. So, let's read the first and last lines of the passage in order to get an idea of the primary purpose. The first line says "Great news for Star Trek fans: warp drives that can propel starships around the Galaxy faster than the speed of light may be possible after all--with a little help from Dr Who." The last line is a quote by a physicist that says "Of course, there are still some basic questions--like how does one go about constructing this Tardis space-time--but it puts the concept of space warps back on the agenda." From both these sentences, we get the idea of space travel, faster than light travel and space warps – maybe this is a discussion of faster than light space travel. Does that match what you have already read? Yes, basically this is a discussion of the theoretical state of play in the area of faster-than-light space travel. Do any of the 5 answer choices match that? Yes – B, even if the wording is somewhat different from how we are wording it, the idea is almost exactly the same. B is the answer.
Another way of getting to the answer is through elimination of obviously incorrect answer choices. We can eliminate A because the author mentions the popular science fiction program Star Trek merely to introduce the idea of faster-than-light travel, and nothing more. C is a stronger possibility because the second paragraph of the passage does discuss some disagreement among physicists about the possibility of creating a warp-drive, but in the same paragraph the theoretical dilemma seems resolved. Moreover, since the author only discusses this in one paragraph, it cannot be the primary purpose of the entire passage. We can eliminate D because the author does not go into detail discussing the uses of space-warping material. And we can discard E because the author does not really go into how the space-warping bubble would work in the real world.

Title Questions
by Stephen Bolton, 20th August, 1999
Title questions are very similar to Main Idea questions, though are less common. Though some of the example paassage we use in this tutorial and in the Practice Section are from the New Scientist, and therefore have titles, the passages in the real GMAT will not have titles. The question might look like this:

"Which of the following titles best summarizes the passage as a whole?"
Strategy:
Treat this as a Main Idea question. A good title sums up the central idea of a passage. Therefore, in order to answer this type of question:

1. Look in the first and last paragraphs for the main idea. Any conclusion words like therefore, thus, so, hence, etc. that you see are most likely introducing the Main Idea/Title. The correct answer will say the same thing as it says in the text, but using different words.
2. Re-read the first line of every passage, and the last line of the first and last paragraphs. This should give you the general structure or outline of the argument, with which you can answer the Title question.
3. Make brief notes – a couple of words- regarding the Title on the text on your scrap paper while you read.
4. After determining the general structure or content of the argument, eliminate answer choices that are too broad or too specific, i.e. answer choices that go beyond the content of the passage, or that deal with content only discussed in one paragraph of the passage.

What would be an appropriate title for this passage?

A)
Constructing The Tardis
B) How To Make Space-Warping Material
C) Bubbles In Space-Time
D) Faster-Than-Light Travel: A Possibility?
E) Debate On The Uses of Space Travel

Explanation
This passage actually already has a title, "Warp Factor One". But we have to look for another title possibility, one that would be most like the Main Idea of the passage. We look at the first and last paragraphs, and since the Main Idea is that researchers now feel that faster-than-light travel maybe more than mere fantasy, we can find the correct answer choice. Does any answer choice corrspond to this idea? Yes- answer D, which is the correct answer.
We can also find the correct answer through elimination. There is nowhere in the passage where it discusses building Dr. Who's Tardis (pity!), so we can eliminate A. Nor does it tell us how to make space-warping material. Eliminate B. While bubbles in space-time are discussed at some length in one of the paragraphs, we cannot say this is the main concern of the passage, and thus should eliminate C. And nowhere are the uses of space travel discussed, so discard E.

Specific Detail or Target questions are probably the most common types of questions, and the easiest to answer. The question might look like this:

"According to the passage,...."
"The passage states that ...."
Strategy
The Specific Detail or Target that we are looking for could be a Line Number, or a Name or Date. Go to the Line Number or Name or Date, and then read several lines above and below it. Find the answer choice that basically says the same thing as in the passage, though usually with different words or word order.


According to the passage, Pfenning and Ford

A)
demonstrated conclusively the impossibility of faster-than-light travel
B) explored the possibility of bubbles that warp space
C) supported the work of Alcubierre
D) work at of the Institute for Theoretical Physics at the Catholic University of Leuven
E) suggested that a warp drive was not physically possible

Explanation
This is a Specific Detail/Target question, and therefore we look for the Name, Line Number, or Date that will help us. In this case, the detail consists of the names Pfenning and Ford. We scan the text, starting from the top of the passage, looking for the names Pfenning and Ford. We find them in only place, at the beginning of the second paragraph. We read a couple of lines above the names, and keep reading until a few lines after the names. It says "But in 1997 Michael Pfenning and Larry Ford at Tufts University in Medford, Massachusetts, apparently killed this ingenious idea by showing that it needed far more than the entire energy content of the Universe to work (This Week, 26 July 1997, p 6)". The line after that says the research of another physicist then resurrected the possibility of FTL travel, negating the implications of the research of Pfenning and Ford. Now we can answer the question. Do any of the answer choices match the information given around the target area? Yes- E.
Let's also eliminate. If we re-read what the passage says about Pfenning and Ford, we can eliminate B, C, and D. None of them are supported by the information in the passage, so let's eliminate all of them without wasting too much time and with a minimum of fuss. A is tougher to eliminate. From the sentence that mention Pfenning and Ford, it seems their work does rule out the possibility of a space-warp drive. But if we read the next line, it says another researcher said it was indeed possible. So the Pfenning and Ford could not have "conclusively" demonstrated the impossibity of the FTL drive.



This is probably the most difficult type of Reading Comprehension problem. The question might look like this:

"It can be inferred that the author makes which of the following assumptions?"
"Which is an assumption underlying the last sentence of the passage?"
"Which of the following, if true, would most strengthen the hypothesis mentioned in lines 17-19?"
"With which of the following statements regarding chaos theory would the author be most likely to agree?"
Strategy:
1. First, treat this type of problem as a Specific Target question. Look for a target in the question, find it in the text, and then look above and below it. Often you do not have to infer very much, the answer remains within the text.
2. If the answer must be inferred and is not stated explicitly within the text, then choose the answer choice that can be inferred or assumed from the information given. Again, you should not have to infer very much – only one or two logical steps removed from the information in the passage.
3. Make sure that the answer choice you decide on does not violate or contradict the Main Idea of the passage - if it does, the answer choice is probably wrong.

It can be inferred that a house with the propeties of the bubble mentioned in the passage

A)
would be larger on the inside than on the outside
B) could move faster than the speed of light
C) might be very energy efficient
D) could move through time
E) would eventually fold in on itself and be destroyed

Explanation
First, let's try to deal with this question as a Specific Target problem. Is there a target in the question? Yes – the bubble. The bubble is first mentioned at the end of the second paragraph, and then discussed at length throughout the third paragraph. Remember, we have to look above and below that target area (as well as read the target area again), so quickly go through the second, third, and first part of the fourth paragraph.
When you are finished, look at the answer choices. Can any of them be inferred from the information given in the target area? Well, we could eliminate C, D, and E for simply not being supported by the information given in the passage. B – maybe, but a house moving through time seems pretty silly. But in the fourth paragraph the author talks about the Tardis, "which looked like a police box but had a spacious interior". Big on the inside, small on the outside. Is that like our house? Yes- answer A. As well, we can choose A because it does not go against or contradict the Main Idea in this case, which if it had, would have made it necessary to eliminate. So choose A.


The question might look like this:

"The author's attitude towards Morgan's theory could best be described as one of ..."
Strategy:
Look for descriptive words, adjectives or adverbs, that could tell you the author's attitude. For example, the words unfortunately or flaw suggest a negative connotation, while strength or valuable emphasize the positive. Make brief notes – a couple of words- regarding the Tone of the text on your scrap paper while you read. Additionally, keep in mind that the author's attitude toward a theory, book, or ethnic group will almost always be respectful, even when somewhat critical.


The author's attitude towards Miguel Alcibierre's theory could best be described as one of
A)
admiration
B) mild skepticism
C) unbridled scorn
D) dismay
E) complete objectivity

Explanation
Since this is a Tone/Attitude question, we must look in the passage for descriptive words that tell us what the author thinks of Alcibierre and his theory. In the second paragraph the author call's Alcibierre's theory "this ingenious idea". This is positive, and the only positive answer choice is A. A is the correct answer. As well, if we could not find the tone so easily, we could also eliminate C and D at the very least, for being too extreme.


The question might look like this:

"Which of the following best describes the organization of the passage?"
"Which of the following best describes the organization of the first paragraph of the passage?"
"One function of the third paragraph is to...."
Strategy:
Re-read the first line of every passage, and the last line of the first and last paragraphs. This should give you the general structure or outline of the argument, with which you can answer the question. Remember to make brief notes about the structure of the text on your scrap paper. If you are looking for the organization of one paragraph, read the first and second sentence of the paragraph. That will give you a rough idea of what is the structure or organization of the paragraph.
Which of the following best describes the organization of the second paragraph of the passage?
A)
Two investigations that support Alcubierre's theory are introduced
B) Possible objections to the uses of the warp drive are present, and then refuted
C) An objection to the practicality of the theory is raised, and then another work is cited to shore up the applicability of the original theory
D) A work of theoretical physics that supports Alcubierre's theory is raised, and then another that refutes it is presented
E) Alcubierre's theory is analyzed by a panel of several eminent physicists
Explanation
Read the first sentence of the paragraph: "But in 1997 Michael Pfenning and Larry Ford at Tufts University in Medford, Massachusetts, apparently killed this ingenious idea by showing that it needed far more than the entire energy content of the Universe to work (This Week, 26 July 1997, p 6)". Then read the second sentence: "Now Chris Van Den Broeck of the Institute for Theoretical Physics at the Catholic University of Leuven, Belgium, has resurrected Alcubierre's proposal". So if we out those two sentences together, and in different words, first the usefulness of Alcubierre's theory is questioned by two researchers, then the theory is validated by yet another researcher. Which of the answer choices is closest to this? C. None of the other answer choices follow the organizational pattern of the paragraph – they reverse it, or are completely dissimilar. C is the only possible answer.


1. Read the whole text of the passage once.
2. Make brief notes about the text on your scrap paper.
3. Remember that the tone or attitude of the passage is usually respectful and moderate, never going to extremes of praise nor criticism.
4. Look out for structural words that tell you the important ideas or transitions in a passage.
5. Go back to the text of the passage for the answers to specific questions.

Reading Comprehension - Test -2

Reading Comprehension

Practice Test No: 02

10-12 minutes

Should we really care for the greatest actors of the past could
we have them before us? Should we find them too different from
our accent of thought, of feeling, of speech, in a thousand minute
particulars which are of the essence of all three? Dr. Doran's
5 long and interesting records of the triumphs of Garrick, and other
less familiar, but in their day hardly less astonishing, players,
do not relieve one of the doubt. Garrick himself, as sometimes
happens with people who have been the subject of much anecdote
and other conversation, here as elsewhere, bears no very distinct
10 figure. One hardly sees the wood for the trees. On the other hand,
the account of Betterton, "perhaps the greatest of English
actors," is delightfully fresh. That intimate friend of Dryden,
Tillatson, Pope, who executed a copy of the actor's portrait by
Kneller which is still extant, was worthy of their friendship;
15 his career brings out the best elements in stage life. The stage
in these volumes presents itself indeed not merely as a mirror of
life, but as an illustration of the utmost intensity of life, in
the fortunes and characters of the players. Ups and downs,
generosity, dark fates, the most delicate goodness, have nowhere
20 been more prominent than in the private existence of those devoted
to the public mimicry of men and women. Contact with the stage,
almost throughout its history, presents itself as a kind of
touchstone, to bring out the bizarrerie, the theatrical tricks
and contrasts, of the actual world.

1. In the expression “One hardly sees the wood for the trees”, the author apparently intends the word trees to be analogous to

A. features of Doran’s language style
B. details learned from oral sources
C. personality of a famous actor
D. detail’s of Garrick’s life
E. stage triumphs of an astonishing player

2. The doubt referred to in line 7 concerns whether

A. the stage personalities of the past would appeal on a personal level to people like the author
B. their contemporaries would have understood famous actors
C. the acting of famous stage personalities would appeal to us today
D. Garrick was as great as he is portrayed
E. historical records can reveal personality

3. Information supplied in the passage is sufficient to answer which of the following questions?

I Who did Doran think was probably the best English actor?
II What did Doran think of Garrick?
III Would the author give a definite answer to the first question posed in the passage?

A. I only
B. II only
C. I and III only
D. II and III only
E. I, II and III





A sanctuary may be defined as a place where Man is passive and
the rest of Nature active. Till quite recently Nature had her
own sanctuaries, where man either did not go at all or only as
a tool-using animal in comparatively small numbers. But now, in
5 this machinery age, there is no place left where man cannot go
with overwhelming forces at his command. He can strangle to
death all the nobler wild life in the world to-day. To-morrow
he certainly will have done so, unless he exercises due
foresight and self-control in the mean time.

10 There is not the slightest doubt that birds and mammals are
now being killed off much faster than they can breed. And it
is always the largest and noblest forms of life that suffer
most. The whales and elephants, lions and eagles, go. The rats
and flies, and all mean parasites, remain. This is inevitable
15 in certain cases. But it is wanton killing off that I am
speaking of to-night. Civilized man begins by destroying
the very forms of wild life he learns to appreciate most when
he becomes still more civilized. The obvious remedy is to begin
conservation at an earlier stage, when it is easier and better
20 in every way, by enforcing laws for close seasons, game preserves,
the selective protection of certain species, and sanctuaries.

I have just defined a sanctuary as a place where man is passive
and the rest of Nature active. But this general definition is too
absolute for any special case. The mere fact that man has to
25 protect a sanctuary does away with his purely passive attitude.
Then, he can be beneficially active by destroying pests and
parasites, like bot-flies or mosquitoes, and by finding antidotes
for diseases like the epidemic which periodically kills off the
rabbits and thus starves many of the carnivora to death. But,
30 except in cases where experiment has proved his intervention to
be beneficial, the less he upsets the balance of Nature the
better, even when he tries to be an earthly Providence.

4. The author implies that his first definition of a sanctuary is

A. totally wrong
B. somewhat idealistic
C. unhelpful
D. indefensible
E. immutable

5. The author’s argument that destroying bot-flies and mosquitoes would be a beneficial action is most weakened by all of the following except

A. parasites have an important role to play in the regulation of populations
B. the elimination of any species can have unpredictable effects on the balance of nature
C. the pests themselves are part of the food chain
D. these insects have been introduced to the area by human activities
E. elimination of these insects would require the use of insecticides that kill a wide range of insects

6. It can be inferred that the passage is

A. part of an article in a scientific journal
B. extracted from the minutes of a nature club
C. part of a speech delivered to an educated audience
D. a speech delivered in a court of law
E. from a polemical article published in a magazine

7. The purpose of the final paragraph is

A. to sum up the main points of the author’s argument
B. to urge a solution to an increasingly pressing problem
C. to qualify the author’s definition of an important term
D. to propose a program
E. to suggest that man should not intervene in natural environments

Reading Comprehension Test - 1

Much has been made of the wealth-creating power of the New Economy. Technological innovation and entrepreneurial activity, proponents of the New Economy point out, have combined to spur the most prolonged economic expansion in American history, and have swelled the ranks of the upper middle classes. A recent Conference Board study, however, suggests that the economic boom of the 1990s has bypassed the poorest segments of the American population. The study reveals that since 1986, a period in which the American economy has expanded by some 30%, the number of fully employed people living in poverty has grown by some 40%, to nearly 3 million Americans.

Apologists of the New Economy are quick to downplay the significance of this disturbing trend. In the second half of the 1990s, they argue, the plight of the poor has begun to improve. Unemployment rates are at a post-war low, the real wages of low-income workers have edged upwards, and from 1993 to 1998 the poverty rate declined from 15.1% to 12.7%. These modest gains notwithstanding, there is no denying that for the economically disadvantaged the current economic boom has not brought the benefits that the boom of the 1960s did: from 1966 to 1978, for example, the poverty rate among full-time workers fell by more than 50%. There can only be one explanation for the fact that the recent boom has not produced similar effects. While the computerization of the American economy has created many high-skilled jobs in the technology industry, a vast number of the new jobs created in the current boom are of the low-skill and low-pay variety. To the people filling these jobs the New Economy has hardly yielded a rich bounty.


In paragraph two, the "apologists" are most characterized by their:

A role in promoting the rights of those on welfare
B regret at missing out on the New Economy
C sorrowful demeanor
D faith in the positive impact of the New Economy
E contrition with regard to poverty in America


According to the passage, what is the principal shortcoming of the New Economy?

A It has not lowered unemployment rates.
B The technological revolution has impersonalized society.
C It requires too much education to succeed in the age of the New Economy.
D It has not in fact increased the overall wealth of society as much as is often assumed.
E Many of the jobs it has created do not pay a good wage.

Which of the following is the most likely occupation of the author of the passage?

A A speechwriter for a politician
B A lobbyist for the computer industry
C A journalist for an investment magazine
D An advocate for the poor
E A disgruntled employee of the Conference Board





#2

The evolution of painting, and cubism in particular, shared with science the common characteristic of drawing upon late nineteenth-century achievements, but, in so doing, of intensifying and transforming them. The result was the overthrow of much of the heritage of the nineteenth and earlier centuries. In certain respects cubism brought to an end artistic traditions that had begun as early as the fifteenth century. At the same time, the cubists created a new artistic tradition that is still alive, for they originated attitudes and ideas that spread rapidly to other areas of culture and that to an important degree underlie artistic thought even today. Cubism first posed, in works of the highest artistic quality, many of the fundamental questions that were to preoccupy artists during the first half of the twentieth century; the historical and aesthetic importance of cubism, therefore, renders it worthy of the most serious attention.

Cubism developed with extraordinary rapidity between the years 1907 and 1914. From 1914 until about 1925 there were a great many artists painting in a cubist mode, but this later phase produced relatively few stylistic innovations that had not been anticipated to some extent during the pre-war years. By the mid-1920s, a crisis emerged in cubism as in European art generally, bringing to an end a period of almost twenty years during which cubism had been the predominant force behind an entire artistic generation.

In its beginnings, however, and until about 1923, cubism was an exclusively Parisian phenomenon, and it probably could not have been born elsewhere, for reasons of history, geography, and culture. No other city in the world in the early years of the twentieth century could boast of a comparable century-long history of outstanding artistic activity; and the relatively central location of Paris in western Europe served only to facilitate the migration of the most gifted young artists and writers from Spain, Italy, Germany, Russia, and the Low Countries toward this cultural mecca. Paris offered them not only the challenge of their most gifted contemporaries, but also its great museums; it offered a tradition of moral and intellectual freedom, and an artistic bohemia in which they could live cheaply at the edge of society without suffering the ostracism inflicted by the bourgeoisie in smaller, more conservative, and less cosmopolitan European cities. In retrospect it is not surprising that, by the early part of the twentieth century, Paris contained an astonishing number of young men of genius, whose presence constituted an intellectual ‘critical mass’ that soon produced a series of revolutionary cultural explosions.

Which of the following was NOT a reason given by the author that Paris became the center of the artistic world in the early twentieth century?

A Paris was centrally located in Western Europe
B Artists were attracted to Paris because of its many museums
C Parisian society was characterized by greater freedom than other European cities
D The bourgeoisie of Paris were wealthy and provided a vast market for young artists to sell their work
E Paris had a century-long tradition of outstanding artistic activity


Which of the following best summarizes the author’s view of the significance of cubism?

A Cubism was a revolutionary movement that transformed art and has continued to influence art up to the present.
B Cubism was a revolutionary movement that transformed art in the early twentieth century but exercised little
influence after the movement waned in the 1920s.
C Cubism, though an important movement, never exercised much influence outside of Paris.
D Cubism was a short-lived fad and doesn’t deserve serious attention from art critics or art historians.
E Cubism inaugurated a social revolution and had far-reaching effects that were felt far beyond the confines of the art world

The word "ostracism" in the third paragraph most nearly means:

A stigmatization
B gentrification
C calumniation
D poverty
E incrimination


Based on this passage it can be concluded that the author believes that:

A Cubism never exercised much influence outside the city of Paris
B The most innovative period of the cubist movement occurred while Cubism was confined to Paris
C Picasso was the greatest of all cubists
D The crisis that emerged in cubism in the 1920s caused a crisis in the rest of the art world
E Cubism had only an ephemeral impact on art


In the third paragraph the artist describes Paris as a "cultural mecca" because:

A he means to suggest that Paris was greatly influenced by Middle Eastern trends
B he believes that Paris’ importance as a religious center resulted in great amounts of artistic patronage
C he believes that Paris’ age-old importance as a center of European art was waning
D he means to suggest that Paris represented a place to which artists flocked from all over Europe
E he means to suggest that the Parisian art world had become decadent



# 3


Ichneumon wasps are a family (Ichneumonidae) of insects in the order Hymenoptera. While the life cycle of most ichneumons is still largely undocumented, observation of some of the known species of the order enables it to relate some established facts. Most ichneumons overwinter in the pupal stage, although a few species overwinter as adult fertilized females. In spring, after emerging from their pupae, the adults mate; the females of the species seek out other insects in which to lay their eggs. Unlike other members of the order Hymenoptera, which includes bees and ants as well as wasps, almost all species of ichneumon wasps are parasitic. Most parasitize only one species; a particular favorite is the moth or butterfly caterpillar. Virtually all the life cycle - barring the adult stage - takes place inside other insects, making observation difficult. It seems, however, that most eggs hatch while the host is still alive; wasp larvae usually live inside the host without killing it. Occasionally, the wasp begins its own development in one stage of its host's own life cycle, and will not mature until the host itself has transformed into another stage.

Adult ichneumons may be observed moving about flowers and leaves, feeling these with their antennae, searching for the prey on which they will parasitize. Many of the larger ichneumons lay their eggs on caterpillars and in the cocoons of large silk moths. If an insect is living and active, one cannot determine by observation alone whether it has been parasitized. However, dryness and brittleness of dead caterpillars and cocoons are often the signs that these had served as hosts to an ichneumon. The presence of small, hard, oval pupal cases inside the remains of the caterpillar or cocoon will corroborate these signs.

Megarhyssa macrurus, one of the largest ichneumons, lays its eggs in the developing larva of the horntail, a primitive wasp whose larva feeds in tunnels inside wood. The female megarhyssa uses her antennae to sense the vibrations of the horntail larvae inside the wood; her ovipositor penetrates through the wood, allowing her to lay her eggs in the horntail larvae. The eggs develop in the larvae, killing them when they are fully grown. The wasp pupates in the horntail tunnel, and chews its way through the bark upon maturity.

The primary purpose of this passage is to:

A Catalogue the destructive behavior of ichneumons.
B Explain what parasitical activity entails.
C Contrast the ways in which ichneumons differ from other hymenoptera.
D Detail what facts are known about the life cycle of ichneumons.
E Register the horntail, silk moth and butterfly among the targets of the ichneumon.


The passage says that ichneumons parasitize which of the following:
I Butterfly caterpillar
II Cocoons of silk moths
III Other wasps


A I only
B II only
C III only
D I & II only
E I, II & III


According to the passage above, the order Hymenoptera includes which of the following:
I Horntail wasps
II Parasitic bees
III All ants


A I only
B I and II
C I, II and III
D II and III only
E I and III


Overwinter, in paragraph one, most likely means:

A to hibernate
B to camouflage oneself
C to metamorphose
D to lower one's body temperature
E to avoid the cold


According to the passage above the life cycle of most ichneumons is still largely undocumented because:

A There has been little interest in the species.
B The bulk of their activity is concealed in inaccessible locations.
C Their tendency to sting makes close-range observation perilous.
D They only exist in geographically remote regions.
E Most ichneumons are so small as to be imperceptible to the naked eye.





# 4

Health scientists have long been puzzled by the so-called French Paradox -- the perplexing fact that the French consume at least as much fat and cholesterol as Americans do, smoke more cigarettes, and yet have far lower rates of coronary heart disease. After a thorough consideration of the two peoples' lifestyles, researchers have isolated yet another distinction between the French and the Americans. The French drink much more wine than do Americans, typically consuming it at meals.

These facts, combined with numerous studies conducted over the past twenty years, point to the fact that light to moderate consumption of alcohol, especially red wine, reduces the rate of heart disease dramatically. Research shows that men and women who drink a glass of wine a day exhibit a 20 to 50 percent lower risk of heart disease, and recent studies suggest that tempered consumption of alcohol reduces the risk of stroke.

Alcohol increases concentrations of HDL, the 'good' cholesterol that lowers the likelihood of heart disease and decreases platelet aggregability: it makes the blood less sticky and less likely to clot, thus decreasing the risk of a heart attack. With its many antioxidant components, including tannins, phenols, resveratrol, and quercitin, in addition to alcohol, red wine appears to be especially protective.


Based on the second paragraph of the passage, we can infer which of the following:

A Red wine is less expensive in France than in America
B More French vineyards produce red wine than white
C The positive effects of wine consumption compound the effects of cigarette smoking
D Of all alcoholic beverages, only red wine has beneficial medicinal effects
E Americans would be healthier if they drank more red wine


The word 'aggregability' in the third paragraph most likely means:

A the tendency to coagulate
B the tendency to oxidize
C the tendency to flatten
D the tendency to protect
E the tendency to dissipate


Based on the third paragraph, we can infer a connection between which of the following elements:

A 'good' cholesterol and 'bad' cholesterol
B oxidation and platelet aggregability
C 'good' cholesterol and blood clotting
D antioxidants and protection from heart disease
E resveratrol and HDL

The first sentence of the passage assumes that:

A The French have a healthier lifestyle than Americans
B There is a link between fat and cholesterol consumption and coronary heart disease
C Cigarette smoking is fashionable in France
D Coronary heart disease is usually fatal
E Levels of second-hand smoke in the air impact rates of coronary heart disease


According to this passage, the key to solving the French Paradox lies in understanding which of the following:

A The relationship between alcohol and HDL
B The relationship between cheese and 'bad cholesterol'
C The relationship between heart disease and stroke
D The relationship between alcohol and nicotine
E The relationship between environmental and genetic factors



# 5

The following text is adapted from the introduction to an 1871 book about Japan written by a British diplomat.

The books which have been written recently about Japan have either been compiled from official records, or have contained the sketchy impressions of passing travelers. Of the inner life of the Japanese the world at large knows but little: their religion, their superstitions, their ways of thought, the hidden springs by which they move—all these are as yet mysteries. Nor is this to be wondered at. The first Western men who came in contact with Japan—I am speaking not of the old Dutch and Portuguese traders and priests, but of the diplomats and merchants of eleven years ago—met with a cold reception. Above all things, the native Government threw obstacles in the way of any inquiry into their language, literature, and history. The fact was that the Tycoon's Government—with whom alone, so long as the Mikado remained in seclusion in his sacred capital at Kyoto, any relations were maintained—knew that the Imperial purple with which they sought to invest their chief must quickly fade before the strong sunlight which would be brought upon it so soon as there should be European linguists capable of examining their books and records. No opportunity was lost of throwing dust in the eyes of the newcomers, whom, even in the most trifling details, it was the official policy to lead astray. Now, however, there is no cause for concealment; the King has shaken off his sloth, and an intelligible Government, which need not fear scrutiny from abroad, is the result: the records of the country being but so many proofs of the Mikado's title to power, there is no reason for keeping up any show of mystery. The path of inquiry is open to all; and although there is yet much to be learnt, some knowledge has been attained, in which it may interest those who stay at home to share.

The recent revolution in Japan has wrought changes social as well as political; and it may be that when, in addition to the advance which has already been made, railways and telegraphs shall have connected the principal points of the Land of Sunrise, the old Japanese, such as he was and had been for centuries when we found him eleven short years ago, will have become extinct. It has appeared to me that no better means could be chosen of preserving a record of a curious and fast disappearing civilization than the translation of some of the most interesting national legends and histories, together with other specimens of literature bearing upon the same subject. Thus the Japanese may tell their own tale, their translator only adding here and there a few words of heading or tag to a chapter, where an explanation or amplification may seem necessary. I fear that the long and hard names will often make my tales tedious reading, but I believe that those who will bear with the difficulty will learn more of the character of the Japanese people than by skimming over descriptions of travel and adventure, however brilliant. The lord and his retainer, the warrior and the priest, the humble artisan and the despised Eta or pariah, each in his turn will become a leading character in my budget of stories; and it is out of the mouths of these personages that I hope to show forth a tolerably complete picture of Japanese society.

Having said so much by way of preface, I beg my readers to fancy themselves wafted away to the shores of the Bay of Yedo—a fair, smiling landscape: gentle slopes, crested by a dark fringe of pines and firs, lead down to the sea; the quaint eaves of many a temple and holy shrine peep out here and there from the groves; the bay itself is studded with picturesque fisher-craft, the torches of which shine by night like glow-worms among the outlying forts; far away to the west loom the goblin-haunted heights of Oyama, and beyond the twin hills of the Hakone Pass --Fujiyama, the Peerless Mountain, solitary and grand, stands in the centre of the plain, from which it sprang vomiting flames twenty-one centuries ago. For a hundred and sixty years the huge mountain has been at peace, but the frequent earthquakes still tell of hidden fires, and none can say when the red-hot stones and ashes may once more fall like rain over five provinces.

In the midst of a nest of venerable trees in Takanawa, a suburb of Yedo, is hidden Sengakuji, or the Spring-hill Temple, renowned throughout the length and breadth of the land for its cemetery, which contains the graves of the Forty-seven. Ronins, famous in Japanese history, heroes of Japanese drama, the tale of whose deeds I am about to transcribe.

And now for the story
.
In Japan, the decade before the writing of this text had been:

A A period of relative political stability.
B A period interrupted by volcanic eruptions.
C A time of social upheaval and political revolution.
D A time when the traditional Japanese roles of warrior, priest, and artisan were being rediscovered.
E A period of unprecedented production of new stories and legends regarding Japanese culture.


The phrase "no opportunity was lost of throwing dust in the eyes of newcomers" describes

A An effort to dazzle visitors with details of the literary and artistic accomplishments of the Japanese.
B The effects of ashes from Mt. Fujiyama.
C The efforts to keep holy shrines private.
D A custom observed since the arrival of Dutch and Portuguese traders.
E The previous government's efforts to deceive foreigners about its legitimacy.


What will follow in the remainder of the book?

A An account of the social changes that had recently occurred in Japan after the revolution.
B An analysis of the contents of the official government records that had previously been supressed.
C A novel by the author depicting the way of life of contemporary Japanese.
D A translation of various stories and legends of traditional Japanese culture.
E A description of the physical features of the Japanese landscape.


The author expects that over the next few years:

A Japan will revert to more traditional ways.
B The Ronins will resume leadership in Japan.
C The ancient class system will be reinstated.
D Modernization will likely transform the traditional social structure.
E International influences in Japan will diminish.


At the time the author wrote, Mount Fujiyama had been:

A A recently active volcano with violent eruptions comparable to the social revolution discussed in the text.
B Dormant for twenty one centuries.
C Dormant for the last 160 years.
D The solitary place of the seclusion of the Mikado.
E Discovered to be only one of a series of 47 other volcanoes.




The politically minded among us may do well to take a cue from the literati in our midst, our colleagues in the fields of language and literature who have contrived to sift through the layers of meaning inherent in language itself, unmasking the power of the metaphor as unpacking the associations of the symbol. As Americans, we take as given the decision made by the United States Congress on June 20, 1798 to adopt the bald eagle as our national emblem. And yet, though it graces our national currency and adorns our federal buildings, even the most reflective of us gives the bird scant thought. We are affronted by the notion of hunting it; we are concerned, if abstractly, that its numbers in the wild continue to decrease. But we pay little, if any mind, to the emblem itself, to the very sign and what it signifies.

Benjamin Franklin, himself a paragon of American virtue, opposed the bird known in his day as the American eagle as the choice for the national emblem. While the logic underlying his objection remains unarticulated, that of American ornithologist Arthur Cleveland Bent, who lived a century after Franklin, was unequivocal. He lamented that the bird's carrion-eating habits and its piratical attacks on the smaller, weaker osprey - itself a bird of prey - "hardly inspire respect and certainly do not exemplify the best in American character." Except, of course, that we have come, as a nation, to uphold the mercenary and the combative. Further fitting in these dubious times, is the fact that our beau idéal not only eats carrion, which it frequently steals from the osprey, but is also a skilled hunter, able to swoop down to capture water fowl in flight and rabbits on the run.

It is the eagle's competence as a hunter, its ability to secure food at will, that allows our eagle to perch for hours, conserving strength where other, inept birds squander their energy searching for food. So, to the charges of viciousness and thievery, we might add indolence: the bald eagle is a sedentary creature that often remains on the same branch or crag for long stretches of time. If we have begun to doubt the prudence, or perhaps more accurately, the desirability of emulating our national bird - of being a nation upholding alternately the virtues of violence, opportunism and laziness - let us conclude with one last comment about the behavior of the bird. When the eggs of the bald eagle hatch, the largest and strongest chick is very aggressive and frequently kills the younger and weaker birds in what is referred to as 'the Cain and Abel battle.' In this nation where the rich continue to prosper as the poor grow poorer, where the strong thrive and the weak languish, it would seem we have begun to perfect an imitative battle.

With which of the following statements would the author of this passage be least likely to agree?

A Americans can be mercenary and opportunistic.
B Americans can be violent.
C The American ideal of equality for all has been realized.
D The flaws of the bald eagle may be evidenced in the behavior of the American population.
E Americans tend toward laziness.


'Carrion' (paragraph 2) most likely means:

A a small bird of the warbler family
B a natural predator
C a flesh-eating animal
D the decaying flesh of a dead animal
E the flesh of caribou and reindeer


When the author states that Arthur Cleveland Bent was "unequivocal", he means to say that Bent was:

A condemning the symbol
B trying to provoke his listeners
C unwilling to compromise
D intending to spark controversy
E clear about his meaning


The author mentions all of the following as negative traits of the Bald Eagle except:

A gluttony
B indolence
C bloodthirst
D thievery
E savagery


The author's primary argument is that:

A The United States needs to select a new bird as its emblem.
B The values of the United States, as epitomized in the bald eagle, can scarcely be considered virtues.
C It is impossible to over-read the meaning of a symbol.
D Poverty in the United States threatens to undermine the quality of American life.
E The inevitable extinction of the bald eagle should not be lamented.




Paul Cezanne (1839-1906) was in many respects the archetypal artist, the tortured soul racked with doubt, filled with conflict. The doubt that engulfed him arose precisely because of the artistic conflict storming within him: in every stroke of the brush he waged the eternal struggle between chaos and order. He debated whether, like Poussin before him, he should create an ordered, harmonious vision of the world. Or should he acknowledge the evidence the world had set before him, that it inherently lacked order, that order was imposed - arbitrarily, even - from without? Could he acknowledge that the order of the world is entirely imposed by its observer, that we as human readers of the world around us discern patterns and impose them upon nature, that we construct meaningful frameworks to link together disparate components? Like the little child who lies in bed, closing one eye and then another, observing with each blink a shift in the room before her, Cezanne became more and more struck by the relativity of vision. It became increasingly significant to him that a slight tilt to the head, a closing of the eye, would alter the appearance of reality.

Cezanne longed to counter the fragmentariness of perceived reality; he sought, by contrast, precision and synthesis. Torn between a desire to represent the world as it is - the room seen with one eye closed and the world then seen with the other shut - and a desire to create a unified representation of the world, Cezanne ultimately arrived at an artistic compromise: the dialectic. He began to paint the variations that the slightly changed viewpoint allowed him to perceive. Rather than dedicate the one tree before him to the canvas, he would paint several possible trees. In his later work, he furthered his means of representing possibility, or uncertainty, by leaving a large portion of his canvas blank. One effect of this innovation was that it encouraged the viewer to envision her own variations of the variations already presented, thereby adding her own perspective to Cezanne's. Nature was no longer laid out before the viewer to be subjected to her scrutiny, rather it includes her, and the evidence of her own senses as part of the constantly changing world. With Cezanne, the viewer becomes part of the view. The consequence of the innovation was extremely significant for Cezanne himself, a first step toward taming the battle within him. By presenting the possibilities, he was able to establish certainty, a certainty paradoxically built on the acceptance of doubt.

Dialectic in paragraph 2 means:

A a reconciliation of opposites
B a logical argumentation
C a reasoned antithesis
D a study over a period of time
E a clear distinction


The author mentions Poussin in the first paragraph primarily in order to:

A Show the artistic tradition out of which Cezanne came.
B Give an example of a painter who influenced Cezanne.
C Offer an example of how radical Cezanne's move away from his predecessors was.
D Denote a painter who sought to order the world in his works.
E Indicate a painter aware of the impossibility of representing reality.


The author understands Cezanne's use of blank space to be:

A A brilliant innovation because it serves to include the viewer in the act of composition.
B A definitive break that allowed Cezanne to free himself from Poussin's influence.
C A necessary step that allowed Cezanne to cure himself of his neuroses.
D A reflection of Cezanne's laziness as a painter.
E An answer to the postmodern questions of fragmentariness and indeterminacy.


The tone of the author's discussion of Cezanne is best described as:

A cautiously enthusiastic
B unconditionally supportive
C blindly laudatory
D unduly deferential
E profoundly respectful


The author's main purpose in this passage is to:

A Argue that the art world would become stale without innovations like Cezanne's.
B Illustrate how childlike whimsy can produce artistic masterpieces.
C Critique those artists, who, unlike Cezanne, sought to order an inherently chaotic world.
D Detail the most significant contribution Cezanne made to art.
E Situate Cezanne among other famous artists.



The young man shut the door with a sharper slam than any visitor had used that afternoon, and walked up the street at a great pace, cutting the air with his walking-stick. He was glad to find himself outside that drawing-room, breathing raw fog, and in contact with unpolished people who only wanted their share of the pavement allowed them. He thought that if he had had Mr. or Mrs. or Miss Hilbery out here he would have made them, somehow, feel his superiority, for he was chafed by the memory of halting awkward sentences which had failed to give even the young woman with the sad, but inwardly ironical eyes a hint of his force. He tried to recall the actual words of his little outburst, and unconsciously supplemented them by so many words of greater expressiveness that the irritation of his failure was somewhat assuaged. Sudden stabs of the unmitigated truth assailed him now and then, for he was not inclined by nature to take a rosy view of his conduct, but what with the beat of his foot upon the pavement, and the glimpse which half-drawn curtains offered him of kitchens, dining- rooms, and drawing-rooms, illustrating with mute power different scenes from different lives, his own experience lost its sharpness.

His own experience underwent a curious change. His speed slackened, his head sank a little towards his breast, and the lamplight shone now and again upon a face grown strangely tranquil. His thought was so absorbing that when it became necessary to verify the name of a street, he looked at it for a time before he read it; when he came to a crossing, he seemed to have to reassure himself by two or three taps, such as a blind man gives, upon the curb; and, reaching the Underground station, he blinked in the bright circle of light, glanced at his watch, decided that he might still indulge himself in darkness, and walked straight on.

And yet the thought was the thought with which he had started. He was still thinking about the people in the house which he had left; but instead of remembering, with whatever accuracy he could, their looks and sayings, he had consciously taken leave of the literal truth. A turn of the street, a firelit room, something monumental in the procession of the lamp-posts, who shall say what accident of light or shape had suddenly changed the prospect within his mind, and led him to murmur aloud:

"She'll do. . . . Yes, Katharine Hilbery'll do. . . . I'll take Katharine Hilbery."

As soon as he had said this, his pace slackened, his head fell, his eyes became fixed. The desire to justify himself, which had been so urgent, ceased to torment him, and, as if released from constraint, so that they worked without friction or bidding, his faculties leapt forward and fixed, as a matter of course, upon the form of Katharine Hilbery. It was marvelous how much they found to feed upon, considering the destructive nature of Denham's criticism in her presence. The charm, which he had tried to disown, when under the effect of it, the beauty, the character, the aloofness, which he had been determined not to feel, now possessed him wholly; and when, as happened by the nature of things, he had exhausted his memory, he went on with his imagination. He was conscious of what he was about, for in thus dwelling upon Miss Hilbery's qualities, he showed a kind of method, as if he required this vision of her for a particular purpose. He increased her height, he darkened her hair; but physically there was not much to change in her. His most daring liberty was taken with her mind, which, for reasons of his own, he desired to be exalted and infallible, and of such independence that it was only in the case of Ralph Denham that it swerved from its high, swift flight, but where he was concerned, though fastidious at first, she finally swooped from her eminence to crown him with her approval. These delicious details, however, were to be worked out in all their ramifications at his leisure; the main point was that Katharine Hilbery would do; she would do for weeks, perhaps for months. In taking her he had provided himself with something the lack of which had left a bare place in his mind for a considerable time. He gave a sigh of satisfaction; his consciousness of his actual position somewhere in the neighborhood of Knightsbridge returned to him, and he was soon speeding in the train towards Highgate.

Based on the information provided in the first paragraph, which of the following best describes the feelings the young man is experiencing as a result of the meeting he just had with the Hilberrys?

A mild displeasure
B warm approval
C temporary indecision
D abject humiliation
E good-humored embarrassment


In the final paragraph, the young man's thoughts are mainly on:

A becoming more oriented to his physical surroundings
B reviewing the events of the earlier interview
C altering a previous impression of Hilberry
D remembering long forgotten aspects of Hilberry
E preparing a rebuttal to Denham on the subject of Hilberry


Which of the following best describes the transformation in the emotional state of the subject of this passage as he walks through the city?

A happiness to confusion
B indecision to abject sadness
C confused embarrassment to extreme anger
D agitation to resolve
E mild indignation to extreme anger


In the last paragraph the author writes that the young man’s faculties became fixed "without friction or bidding" on Katherine Hilberry. Which of the following best restates the author's meaning:

A the man’s thoughts unconsciously turned to Katherine Hilberry
B the man suddenly saw Katherine Hilberry in the lamplight
C the man’s thoughts towards Katherine Hilberry became less antagonistic
D the man stealthily approached Katherine Hilberry
E the man began replaying a pleasant conversation with Hilberry in his head


The word "chafed" as used in the first paragraph most nearly means:

A annoyed
B scraped
C made impatient
D warmed
E reminded





But man is not destined to vanish. He can be killed, but he cannot be destroyed, because his soul is deathless and his spirit is irrepressible. Therefore, though the situation seems dark in the context of the confrontation between the superpowers, the silver lining is provided by amazing phenomenon that the very nations which have spent incalculable resources and energy for the production of deadly weapons are desperately trying to find out how they might never be used. They threaten each other, intimidate each other and go to the brink, but before the total hour arrives they withdraw from the brink.

1. The main point from the author's view is that
1. Man's soul and spirit can not be destroyed by superpowers.
2. Man's destiny is not fully clear or visible.
3. Man's soul and spirit are immortal.
4. Man's safety is assured by the delicate balance of power in terms of nuclear weapons.
5. Human society will survive despite the serious threat of total annihilation.


2. The phrase 'Go to the brink' in the passage means
1. Retreating from extreme danger.
2. Declare war on each other.
3. Advancing to the stage of war but not engaging in it.
4. Negotiate for peace.
5. Commit suicide.

3. In the author's opinion
1. Huge stockpiles of destructive weapons have so far saved mankind from a catastrophe.
2. Superpowers have at last realized the need for abandoning the production of lethal weapons.
3. Mankind is heading towards complete destruction.
4. Nations in possession of huge stockpiles of lethal weapons are trying hard to avoid actual conflict.
5. There is a Silverlining over the production of deadly weapons.


4. 'Irrepressible' in the second line means
1. incompatible
2. strong
3. oppressive
4. unrestrainable
5. unspirited


5. A suitable title for the above passage is
1. Destruction of mankind is in evitable.
2. Man's desire to survive inhibits use of deadly weapons.
3. Mounting cost of modern weapons.
4. Threats and intimidation between super powers.
5. Cowardly retreat by man





Disequilibrium at the interface of water and air is a factor on which the transfer of heat and water vapor from the ocean to the air depends. The air within about a millimeter of the water is almost saturated with water vapor and the temperature of the air is close to that of the surface water. Irrespective of how small these differences might be, they are crucial, and the disequilibrium is maintained by air near the surface mixing with air higher up, which is typically appreciably cooler and lower in water vapor content. The turbulence, which takes its energy from the wind mixes the air. As the speed of wind increases, so does the turbulence, and consequently the rate of heat and moisture transfer. We can arrive at a detailed understanding of this phenomenon after further study. The transfer of momentum from wind to water, which occurs when waves are formed is an interacting-and complicated phenomenon. When waves are made by the wind, it transfers important amounts of energy-energy, which is consequently not available for the production of turbulence.

1. This passage principally intends to:
1. resolve a controversy
2. attempt a description of a phenomenon
3. sketch a theory
4. reinforce certain research findings
5. tabulate various observations


2. The wind over the ocean usually does which of the following according to the given passage?
I. Leads to cool, dry air coming in proximity with the ocean surface.
II. Maintains a steady rate of heat and moisture transfer between the ocean and the air.
III. Results in frequent changes in the ocean surface temperature.
1. I only
2. II only
3. I and II only
4. II and III only
5. I, II, and III


3. According to the author the present knowledge regarding heat and moisture transfer from the ocean to air as
1. revolutionary
2. inconsequential
3. outdated
4. derivative
5. incomplete


4. According to the given passage, in case the wind was to decrease until there was no wind at all, which of the following would occur?
1. The air, which is closest to the ocean surface would get saturated with water vapor.
2. The water would be cooler than the air closest to the ocean surface.
3. There would be a decrease in the amount of moisture in the air closest to the ocean surface.
4. There would be an increase in the rate of heat and moisture transfer.
5. The temperature of the air closest to the ocean and that of the air higher up would be the same.





The Food and Drug Administration has formulated certain severe restrictions regarding the use of antibiotics, which are used to promote the health and growth of meat animals. Though the different types of medicines mixed with the fodder of the animals kills many microorganisms, it also encourages the appearance of bacterial strains, which are resistant to anti-infective drugs.

It has already been observed that penicillin and the tetracyclines are not as effective therapeutically as they once used to be. This resistance to drugs is chiefly caused due to tiny circlets of genes, called plasmids, which are transferable between different species of bacteria. These plasmids are also one of the two kinds of vehicles on which molecular biologists depend on while performing gene transplant experiments. Existing guidelines also forbid the use of plasmids, which bear genes for resistance to antibiotics, in the laboratories. Though congressional dabate goes on as to whether these restrictions need to be toughened with reference to scientists in their laboratories, almost no congressional attention is being paid to an ill advised agricultural practice, which produces deleterious effects.

1. In the present passage, the author's primary concern is with:
1. The discovery of methods, which eliminate harmful microorganisms without generating drug-resistant bacteria.
2. Attempting an explanation of the reasons for congressional inaction about the regulation of gene transplant experiments.
3. Portraying a problematic agricultural practice and its serious genetic consequences
4. The verification of the therapeutic ineffectiveness of anti-infective drugs
5. Evaluation of the recently proposed restrictions, which are intended to promote the growth of meat animals.


2. As inferred from the above passage, the mutual transfer of plasmids between different bacteria can result in which of the following?
1. Microorganisms, which have an in-built resistance to drugs
2. Therapeutically useful circlets of genes
3. Penicillin like anti-infective drugs
4. Viruses used by molecular biologists
5. Carriers for performing gene transplant experiments.

3. According to the above passage the author believes that those who favor the stiffening of restrictions on gene transplant research should logically also.
1. Approve and aid experiments with any plasmids except those, which bear genes for antibiotic resistance.
2. Inquire regarding the addition of anti-infective drugs to livestock feeds
3. Oppose the using of penicillin and tetracyclines in order to kill microorganisms
4. Agree to the development of meatier live-stock through the use of antibiotics
5. Approve of congressional debate and discussion regarding science and health issues.


4. The attitude the author has with reference to the development of bacterial strains that render antibiotic drugs in effective can best be described as
1. indifferent
2. perplexed
3. pretentious
4. insincere
5. apprehensive




Roger Rosenblatt's book Black Fiction, manages to alter the approach taken in many previous studies by making an attempt to apply literary rather than sociopolitical criteria to its subject. Rosenblatt points out that criticism of Black writing has very often served as a pretext for an expounding on Black history. The recent work of Addison Gayle's passes a judgement on the value of Black fiction by clearly political standards, rating each work according to the ideas of Black identity, which it propounds.

Though fiction results from political circumstances, its author react not in ideological ways to those circumstances, and talking about novels and stories primarily as instruments of ideology circumvents much of the fictional enterprise. Affinities and connections are revealed in the works of Black fiction in Rosenblatt's literary analysis; these affinities and connections have been overlooked and ignored by solely political studies.

The writing of acceptable criticism of Black fiction, however, presumes giving satisfactory answers to a quite a few questions. The most important of all, is there a sufficient reason, apart from the racial identity of the authors, for the grouping together of Black authors? Secondly, what is the distinction of Black fiction from other modern fiction with which it is largely contemporaneous? In the work Rosenblatt demonstrates that Black fiction is a distinct body of writing, which has an identifiable, coherent literary tradition. He highlights recurring concerns and designs, which are independent of chronology in Black fiction written over the past eighty years. These concerns and designs are thematic, and they come form the central fact of the predominant white culture, where the Black characters in the novel are situated irrespective of whether they attempt to conform to that culture or they rebel against it.

Rosenblatt's work does leave certain aesthetic questions open. His thematic analysis allows considerable objectivity; he even clearly states that he does not intend to judge the merit of the various works yet his reluctance seems misplaced, especially since an attempt to appraise might have led to interesting results. For example, certain novels have an appearance of structural diffusion. Is this a defeat, or are the authors working out of, or attempting to forge, a different kind of aesthetic? Apart from this, the style of certain Black novels, like Jean Toomer's Cane, verges on expressionism or surrealism; does this technique provide a counterpoint to the prevalent theme that portrays the fate against which Black heroes are pitted, a theme usually conveyed by more naturalistic modes of expressions?

Irrespective of such omissions, what Rosenblatt talks about in his work makes for an astute and worthwhile study. His book very effectively surveys a variety of novels, highlighting certain fascinating and little-known works like James Weldon Johnson's Autobiography of an Ex-Coloured Man. Black Fiction is tightly constructed, and levelheaded and penetrating criticism is exemplified in its forthright and lucid style.

1. The author of the passage raises and objection to criticism of Black fiction like that by Addison Gayle as it:
1. Highlights only the purely literary aspects of such works
2. Misconceive the ideological content of such fiction
3. Miscalculate the notions of Black identity presented in such fiction
4. Replaces political for literary criteria in evaluating such fiction
5. Disregards the reciprocation between Black history and Black identity exhibited in such fiction.


2. The primary concern of the author in the above passage is:
1. Reviewing the validity of a work of criticism
2. Comparing various critical approaches to a subject
3. Talking of the limitations of a particular kind of criticism
4. Recapitulation of the major points in a work of criticism
5. Illustrating the theoretical background of a certain kind of criticism.


3. The author is of the opinion that Black Fiction would have been improved had Rosenblatt:
1. Undertaken a more careful evaluation of the ideological and historical aspects of Black Fiction
2. Been more objective in his approach to novels and stories by Black authors
3. Attempted a more detailed exploration of the recurring themes in Black fiction throughout its history
4. Established a basis for placing Black fiction within its own unique literary tradition
5. Calculated the relative literary merit of the novels he analyzed thematically.


4. Rosenblatt's discussion of Black Fiction is :
1. Pedantic and contentious
2. Critical but admiring
3. Ironic and deprecating
4. Argumentative but unfocused
5. Stilted and insincere.


5. According to the given passage the author would be LEAST likely to approve of which among the following?
1. Analyzing the influence of political events on the personal ideology of Black writers
2. Attempting a critical study, which applies sociopolitical criteria to the autobiographies of Black authors
3. A literary study of Black poetry that appraises the merits of poems according to the political acceptability of their themes
4. Studying the growth of a distinct Black literary tradition within the context of Black history
5. Undertaking a literary study, which attempts to isolate aesthetic qualities unique to Black fiction.


6. From the following options, which does the author not make use of while discussing Black Fiction?
1. Rhetorical questions
2. Specific examples
3. Comparison and contrast
4. Definition of terms
5. Personal opinion.


7. The author makes a reference to James Weldon Johnson's Autobiography of an Ex-colored Man most probably to:
1. Highlight the affinities between Rosenblatt's method of thematic analysis and earlier criticism
2. Elucidate regarding the point made regarding expressionistic style earlier in the passage
3. Qualify the assessment of Rosenblatt's book made in the first paragraph of the passage
4. Demonstrate the affinities among the various Black novels talked of by Rosenblatt's literary analysis
5. Present a specific example of one of the accomplishments of Rosenblatt's work.




Some modern anthropologists hold that biological evolution has shaped not only human morphology but also human behavior. The role those anthropologists ascribe to evolution is not of dictating the details of human behavior but one of imposing constraints - ways of feeling, thinking, and acting that ''come naturally'' in archetypal situations in any culture. Our ''frailties'' - emotions and motivs such as rage, fear, greed, gluttony, joy,lust, love-may be a very mixed assortment quality: we are, as we say, ''in the grip'' of them. And thus they give us oursense of constraints.

Unhappily, some of those frailties our need for ever-increasing security among them are presently maladaptive. Yet beneath the overlay of cultural detail, they, too, are said to be biological in direction, and therefore as natural to us as are our appendixes. We would need to comprehend throughly their adaptive origins in order to understand how badly they guide us now. And we might then begin to resist their pressure.

1. The author implies that control to any extent over the ''frailties'' that constrain our behavior is though to presuppose
1. That those frailties and adaptive are recognized as currently beneficial and adaptive
2. That there is little or no overlay of cultural detail that masks their true nature.
3. That there are cultures in which those frailties do not ''come naturally'' and from which such control can be learned
4. A full understanding of why those frailties evolved and of how they function now
5. A thorough grasp of the principle that cultural detail in human behavior can differ arbitrarily from society to society.


2. It can be inferred that in his discussion of maladaptive frailties the author assumes that
1. Evolution does not favor the emergence of adaptive characteristics over the emergence of maladaptive ones
2. Any structure or behavior not positively adaptive is regarded as transitory in evolutionary theory
3. Maladaptive characteristics, once fixed, make the emergence of other maladaptive characteristics more likely
4. The designation of a characteristic as being maladaptive must always remain highly tentative
5. Changes in the total human environment can outpace evolutionary change.

3. The primary purpose of the passage is to present
1. A position on the foundations of human behavior and on what those foundations imply
2. A theory outlining the parallel development of human morphology and of human behavior
3. A diagnostic test for separating biologically determined behavior patters from culture - specific detail
4. An overview of those human emotions and motive's that impose constraints on human behaviour
5. A practical method for resting the pressures of biologically determined drives.


4. Which of the following most probably provides an appropriate analogy from human morphology for the ''details'' versus ''constraints'' distinction made in the passage in relation to human behaviour?
1. The ability of most people to see all the colors of the visible spectrum as against most peoples inability to name any but the primary colors
2. The ability of even the least fortunate people to show compassion as against people's inability to mask their feelings completely
3. The ability of some people to dive to great depths as against most people's inability to swim long distance
4. The psychological profile of those people who are able to delay gratification as against people's inability to control their lives completely
5. The greater lung capacity of mountain peoples that helps them live in oxygen-poor air as against people's inability to fly without special apparatus.